Section 10 Neurology
SECTION 10 Neurology
793
CHAPTER 40
794
Headache
795 Case: A 50-year-old man with tinnitus A previously healthy 50-year-old man is evaluated in the clinic for infectious mononucleosis or viral labyrinthitis. However, his have time. Recently, the patient has developed ringing in the ears. Hever 6 months of fatigue, headache, and intermittent fever. He was previously evaluated for these symptoms and was thought to symptoms are progressing, and he is seeking another opinion. The headache is diffuse and constant and has become more intense o estimates weight loss of 15 pounds since the symptoms began. The patient lives with his wife in Portland, Oregon. Over the past few
years, he has visited Morocco, British Columbia, Texas, and California. Temperature is 37.5°C. The patient is alert and oriented. The sinuses He has no pets. are nontender. Fundoscopic examination reveals bilateral papilledema. There are no focal neurologic abnormali ies on examination. Computed tomography (CT) imaging oftthe brain is unremarkable. Human immunode iciency virus (HIV) antibodies are not detecte
in the serum. Lumbarfpuncture reveals the following cerebrospinal d fluid (CSF) profile:
Appearance Pressure (cm H2O) WBC/µL Glucose (mg/dL) Total protein (mg/dL) CleaMagnetic resonance imaging0(MRI) of the brain with axial (Figure 1 ×1.2 cm right cerebellar mass (arrow) and diffuse leptomeningeal r 42 84 (72% lymphocytes) 17 21 40-1A) and sagittal (Figure 40-1B) views reveals a ring-enhancing nodularity.
FIGURE 40-1
What is the most likely cause of headache in this patient?
How common is headache in the general population? What are the 2 general categories of headache
Headache is one of the most common presenting complaints in the primary care clinic, affecting the vast majority of men and women at some point during life.1,2
Headache disorders can be primary or secondary.
796
disorders?
Are primary or Primary disorders account for most cases of headache, particularly in the primary care clinic. However, the frequency of secondary secondary headache increases in certain populations (eg, immunocompromised patients, patients with a history of headaches malignancy).1,2 more common? What are the Secondary headache disorders are suggested by the presence of any of the following features: sudden- or new-onset clinical clues headache at an older age (>40 years), change in the quality of chronic headache, systemic symptoms or illness (eg, fever, to the presence history of malignancy, nausea or vomiting, neck stiffness, immunocompromised status, use of anticoagulant of a secondary medications), neurologic symptoms or signs (eg, confusion, focal neurologic deficits, seizure, papilledema), headaches headache that wake patients from sleep, and headaches that are worsened with Valsalva maneuvers.3 disorder?
797
Primary Headache Disorders
What are the primary headache disorders?
Bilateral, mild in intensity, “tightening” or “pressing” in quality, often associated with stress, and lasting for minutes to days. A 23-year-old woman with an 8-month history of recurring unilateral pulsatile headache lasting anywhere from a few hours to a few days, associated with nausea, and typically preceded by visual disturbance. This primary headache disorder is more common in men and is characterized by unilateral retro-orbital pain associated with lacrimation, conjunctival erythema, and nasal drainage.
What are the pharmacologic treatment options for tension headaches? What are the characteristics of migraine headaches?
How often are migraines associated with aura?
What are the pharmacologic treatment options for migraine headaches?
Cluster headaches are part of what group of primary headache disorders?
What other conditions belong to the trigeminal autonomic cephalalgias?
How common are cluster headaches? What are the pharmacologic treatment options for cluster headaches?
Tension headache.2
Migraine headache.
Cluster headache.2
Acetaminophen or nonsteroidal anti-inflammatory drugs (NSAIDs) are generally effective for tension headaches.2 Migraine headaches are typically unilateral, pulsatile, moderate to severe in intensity, aggravated by routine physical activity, and often associated with nausea, photophobia, or phonophobia. Migraines may be preceded by aura, which is a reversible focal neurologic symptom that develops over 5 to 20 minutes and lasts for <60 minutes.2 Aura occurs in up to one-fifth of patients with migraine (ie, “classic” migraine) and is most commonly visual in nature. The headache tends to be similar in quality to migraine without aura (ie, “common” migraine) but may lack some features. Aura can occur without an associated headache in some cases.2 Abortive and preventive medications are the 2 main approaches to treating migraines. First-line abortive therapies for mild to moderate migraines include NSAIDs or caffeine-containing combination analgesics, whereas triptans and ergotamines are first-line for moderate to severe migraines. There are multiple options for preventive therapies, including antiepileptic drugs (eg, topiramate), β-blockers, calcium channel blockers, and some antidepressants (eg, tricyclics). The choice of therapy should be guided by patient-specific factors.4,5 Cluster headaches are considered one of the trigeminal autonomic cephalalgias (TACs), a group of primary headache disorders characterized by the presence of unilateral pain in the distribution of the trigeminal nerve associated with ipsilateral cranial autonomic features (eg, ptosis, conjunctival injection, lacrimation, nasal congestion, rhinorrhea).6 The TACs include cluster headaches, paroxysmal hemicranias, hemicranias continua, short-lasting unilateral neuralgiform headache attacks with conjunctival injection and tearing (SUNCT), and short-lasting unilateral neuralgiform headache attacks with cranial autonomic features (SUNA).6 Cluster headaches, which occur predominantly in men, are the most common of the TACs but are rare overall (affecting approximately 0.1% of the general population).6 Like migraine headaches, there are abortive and preventive approaches to managing cluster headaches. For acute termination of a cluster headache, nonoral routes of administration are required. Subcutaneously administered sumatriptan is a first-line option. Oxygen inhalation via a nonrebreather facemask (10-15 L/min for 15-20 minutes) is also considered to be first-line abortive therapy. For prevention, verapamil is considered the drug of choice, but there are other options (eg, topiramate, sodium valproate, melatonin, lithium).6
798
Secondary Headache Disorders
What are the 2 anatomic subcategories of secondary headache disorders?
The intracranial causes of secondary headache can be separated into which general subcategories?
Secondary headache disorders can be caused by intracranial or extracranial processes.
The intracranial causes of secondary headache can be separated into the following subcategories: vascular, infectious, tumor, cerebrospinal fluid, and other.
799
Intracranial Vascular Causes of Headache
What are the intracranial vascular causes of headache?
Arteriovenous nicking, Severe systemic hypertension. flame-shaped hemorrhages, cotton wool spots, and papilledema may be present on fundoscopic examination. A 72-year-old man with Posterior circulation stroke. vascular disease presents with acute-onset vertigo, ataxia, and homonymous hemianopsia. A 24-year-old woman with Cerebral venous thrombosis. homozygous factor V Leiden presents with sudden-onset headache after recently starting estrogen-based contraception. This acute vascular event is a Arterial dissection. common cause of stroke in young patients and may occur within the anterior or posterior circulations. Nosebleeds, hemoptysis, and Arteriovenous malformation (AVM) related to hereditary hemorrhagic telangiectasia. cutaneous telangiectasias (Figure 40-2).
FIGURE 40-2 Multiple telangiectasias involving the mucosa of the lower lip in a patient with hereditary hemorrhagic telangiectasia.
Headache in a patient with polycystic kidney disease. “Thunderclap” headache in a patient with polycystic kidney disease. Treatment for this vascular condition generally includes immunosuppressive medications. A mimic of primary angiitis of the central nervous system; this entity usually presents with “thunderclap” headache (which is unusual in primary angiitis of the central nervous system). This vascular condition is most common in Asian populations, with a bimodal age distribution of children around 5 years of age and adults 40 to 50 years of age.
Cerebral aneurysm. Subarachnoid hemorrhage from ruptured cerebral aneurysm.
Vasculitis, including primary angiitis of the central nervous system (PACNS), systemic vasculitis (eg, Behçet’s disease), and vasculitis associated with systemic disease (eg, systemic lupus erythematosus).
Reversible cerebral vasoconstriction syndrome (RCVS).7
Moyamoya disease.8
800
Under what conditions can systemic hypertension cause headache?
What are the characteristics of headache associated with chronic systemic hypertension?
How often is headache associated with stroke?
How common is headache in patients with cerebral venous thrombosis?
Which 2 major intracranial vessels are most commonly involved in arterial dissection? What are the 4 general types of congenital cerebral vascular malformation? What are the characteristics of the headache associated with nonruptured cerebral aneurysms? What are the general types of intracranial hemorrhage? What diagnostic studies are helpful in the diagnosis of primary angiitis of the central nervous system? What is the prognosis of reversible cerebral vasoconstriction syndrome?
What are the characteristics of the headache associated with moyamoya disease?
Headache can occur in association with acute systemic hypertension, where an abrupt rise in blood pressure within the range of autoregulation results in Increased transmural pressure in the larger cerebral arteries, which are sensitive to pain; hypertensive encephalopathy, where an abrupt rise in blood pressure outside the range of autoregulation results in extravasation of plasma and erythrocytes into brain tissue; and chronic systemic hypertension.9 The headache of chronic systemic hypertension is typically bilateral and located in the back of the head. It tends to be nocturnal, present on waking in the morning, and improved on rising. This pattern occurs because at night, when blood pressure generally tends to be lower, patients with chronic systemic hypertension experience a state of relative hypotension, which triggers compensatory cerebral vasodilation via autoregulation, leading to Increased cerebral blood flow and headache. On rising, the erect position increases blood pressure, reversing vasodilation and improving the headache.9,10 Headache occurs in around one-third of patients with stroke (it also occurs in some patients with transient ischemic attack), and usually localizes to the affected hemisphere. It occurs more commonly in patients with posterior circulation stroke where it often localizes to the occipital region.11 Headache occurs in most patients with cerebral venous thrombosis and is often the initial symptom. Associated manifestations are frequent and include papilledema, focal neurologic deficits, seizure, drowsiness, and confusion. Common causes include infection (eg, otitis media), pregnancy, oral contraceptive use, underlying malignancy, connective tissue disease (eg, Behçet’s disease), and thrombophilia (eg, antithrombin III deficiency). CT imaging of the brain is often unrevealing. MRI of the brain combined with magnetic resonance venography is the diagnostic method of choice. Treatment options include anticoagulation, thrombolysis, and surgery.11 The carotid and vertebral arteries are the intracranial vessels most often associated with arterial dissection. Carotid artery dissection is relatively more common.11
The 4 types of congenital cerebral vascular malformation are developmental venous anomaly, capillary telangiectasia, cavernous malformation, and arteriovenous malformation. The headache associated with nonruptured cerebral aneurysms can be acute or chronic. When acute, it is sudden in onset and severe in intensity, comparable to that of subarachnoid hemorrhage. Other clinical manifestations include facial pain, vision loss, seizure, and cranial neuropathy.12 There are 5 main types of intracranial hemorrhage: epidural, subdural, subarachnoid, intraparenchymal, and intraventricular. Headache can occur with any of these entities. CSF evaluation and neuroimaging are helpful in the workup of PACN. The CSF profile is abnormal in most patients and characterized by modest lymphocytic pleocytosis, elevated protein, and the occasional presence of oligoclonal bands. Brain biopsy is ultimately required to make the diagnosis in most cases (it also has the advantage of ruling out mimics such as infection or malignancy).7 RCVS occurs most often in women with a mean age of onset of 42 years. It is caused by transient dysregulation of cerebral vascular tone. Underlying triggers include recent pregnancy, exposure to vasoactive substances (eg, cocaine), and sexual intercourse. The headache is sudden-onset and severe (ie, thunderclap) and is frequently associated with nausea, vomiting, photophobia, confusion, and blurred vision. Most patients with RCVS recover without disability, but stroke with long-term deficits and death have been reported.13 The headache of moyamoya disease is similar to migraine in quality but generally does not respond to pharmacologic therapy. Neuroimaging is required to make the diagnosis (see Figure 42-3). Headache subsides in some patients within a year of surgical treatment, but it is persistent in others.8
801
Intracranial Infectious Causes of Headache
What are the intracranial infectious causes of headache?
A 19-year-old military recruit presents with headache, neck stiffness, and photophobia. Similar to meningitis, but this condition is associated with neurocognitive dysfunction early in the course of illness and can be associated with a normal CSF profile. This condition typically requires surgical drainage and a long duration of intravenous antibiotics. A 24-year-old woman with active intravenous drug use is admitted with aortic valve endocarditis and develops new headache and focal neurologic deficits. Common in patients with acquired immunodeficiency syndrome (AIDS) as a result of reactivation of previously acquired infection, often through exposure to feline feces.
Which of the following cerebrospinal fluid profiles are characteristic of these conditions: normal, viral meningitis, typical bacterial meningitis, atypical bacterial meningitis, fungal meningitis, and aseptic meningitis?
Meningitis. Encephalitis.
Brain abscess.
Cerebral septic emboli.
Cerebral toxoplasmosis.
Fluid type A corresponds to normal CSF; type B to typical bacterial meningitis; type C to fungal meningitis (and some types of atypical bacterial meningitis, eg, tuberculosis); and type D to viral meningitis, most types of atypical bacterial meningitis, and most types of aseptic meningitis. Fluid types C and D may be characterized by a predominance of neutrophils early in the course of illness.14
Fluid type A B C D
WBC/ µL <5 500- 20,000 25-500 5-1000
WBC type Lymphocytes Neutrophils Lymphocytes or neutrophils Lymphocytes
Glucose (mg/dL) 40-80 <40 <40 40-80
Protein (mg/dL) <50 100-700 50-500 <100
What finding may be present on electroencephalography (EEG) in patients with herpes simplex encephalitis? What are the 3 general mechanisms of brain abscess development? Is the risk of cerebral septic emboli limited to left-sided Infective endocarditis?
In patients with human immunodeficiency virus (HIV) infection, what peripheral CD4 cell count is associated with a significant increase in risk of cerebral toxoplasmosis?
In most patients with herpes simplex encephalitis, EEG reveals a temporal focus demonstrating periodic lateralizing epileptiform discharges (ie, lateralized periodic discharges).15 Brain abscess can occur as a result of direct extension from extracranial infection (eg, dental abscess), hematogenous spread, or direct inoculation following head injury or neurosurgery. Neuroimaging is required to make the diagnosis. Paradoxical systemic septic emboli, including cerebral emboli, can occur in patients with right-sided endocarditis and coexistent right-to-left shunt. In patients with patent foramen ovale or other intracardiac shunt, transient right-to-left shunting can occur when right-sided pressures are abruptly Increased (eg, during Valsalva).16 A CD4 cell count <200/µL is associated with a significantly Increased risk of cerebral toxoplasmosis and should trigger initiation of specific prophylactic medication (eg, trimethoprim-sulfamethoxazole).17
802
803
Headache Related to Intracranial Tumor
How common are intracranial tumors? How common is headache in patients with intracranial tumors? What are the characteristics of the headache associated with intracranial tumors? What is the study of choice to evaluate for the presence of an intracranial tumor? What are the 2 general types of intracranial tumors?
How common are metastatic brain tumors?
Which types of cancers are most commonly associated with brain metastases? What is the prognosis of patients with intracranial metastases? What is the most common type of primary intracranial tumor in adults? What is the prognosis of glioblastoma multiforme?
In the industrialized world, the incidence of intracranial tumor is approximately 12 per 100,000 persons.18 Headache occurs in about one-half of patients with intracranial tumors. Other manifestations include nausea, vomiting, seizure, focal neurologic deficits, and neurocognitive dysfunction.18 The headache associated with intracranial tumors is usually bifrontal but worse on the side of the tumor, similar to tension headache in quality, worse with bending over, associated with nausea or vomiting, and can be more noticeable in the morning.18,19 MRI of the brain is the study of choice to evaluate for intracranial tumor. CT imaging can miss lesions of the posterior fossa or nonenhancing tumors (eg, low-grade gliomas).18 Intracranial tumors can be metastatic or primary.
Brain metastases are the most common type of intracranial metastases and are increasing in frequency (likely due to increasing survival rates), occurring in close to one-half of adult cancer patients according to autopsy reports.20 Lung cancer accounts for more than one-half of all cases of brain metastases. Breast cancer, melanoma, renal cell carcinoma, and colorectal cancer make up most of the other cases.20 The prognosis associated with the presence of intracranial metastasis varies according to patient and tumor characteristics but, overall, is quite poor. Virtually all cases are associated with fatal outcomes.21 Gliomas are the most common type of primary intracranial tumor, accounting for more than one-half of all cases. Glioblastoma multiforme is the most frequent subtype.22 The prognosis of glioblastoma multiforme is poor, with a median survival on the order of months and few survivors at 5 years after diagnosis.22
804
Headache Related to Cerebrospinal Fluid
What are the causes of headache related to cerebrospinal fluid?
A 23-year-old obese woman with chronic daily headache is Pseudotumor cerebri (ie, idiopathic intracranial hypertension). found to have papilledema (Figure 40-3), normal blood pressure, normal neuroimaging, and an opening pressure of 48 cm H2O on lumbar puncture with an otherwise normal CSF profile.
FIGURE 40-3 Chronic papilledema secondary to Increased
Accumulation of CSF related to a disturbance in formation, flow, or absorption. May be associated with clear drainage from the nose or ear.
What is the most severe sequela of pseudotumor cerebri?
What are the causes of acquired hydrocephalus?
What quality is highly characteristic of the headache associated with low cerebrospinal fluid pressure?
intracranial pressure. There is optic disc edema with indistinct disc margins and obscured blood vessels. Foundations of Medicine. 5th ed. Philadelphia, PA: Lippincott Williams & (From Rubin R, Strayer DS. Rubin’s Pathology: Clinicopathologic Wilkins; 2008.)
Hydrocephalus.23 Spontaneous cerebrospinal fluid leak.
Headache is present in most patients with pseudotumor cerebri. Associated symptoms include blurred vision and visual loss. Permanent vision loss occurs in a significant number of patients. Systemic hypertension is a major risk factor for the development of vision loss in these patients.24 Acquired hydrocephalus can occur as a result of tumors and cysts (eg, colloid cyst), inflammation (eg, meningitis), and absorption blockages (eg, intracranial hemorrhage, normal pressure hydrocephalus).23 The headache related to CSF leak is typically positional, worse in the upright position and improved with recumbence. Lumbar puncture is a more common cause of low CSF pressure than spontaneous CSF leak.
805
Other Intracranial Causes of Headache
What are the other intracranial causes of headache?
Recurrent episodes of sudden-onset, brief, unilateral, and stabbing pain in the distribution of 1 or more branches of the fifth cranial nerve. A college football player complains of chronic daily headache after sustaining a concussion during a game 6 months ago.
What is first-line pharmacologic treatment for trigeminal neuralgia?
How common is post-traumatic headache after traumatic brain injury (TBI)?
Trigeminal neuralgia.
Post-traumatic headache.
Trigeminal neuralgia is a clinical diagnosis, based on characteristic paroxysms of pain in the distribution of one or more branches of the fifth cranial nerve. Carbamazepine is considered first-line pharmacologic treatment. Oxcarbazepine may be an effective alternative agent.25 At 1 year after TBI, almost one-half of patients experience post-traumatic headache. Women and patients with a baseline headache disorder pre-TBI are more likely to experience post- traumatic headache. Incidence appears to be unrelated to the severity of TBI.26
806
Extracranial Causes of Headache
What are the extracranial causes of headache?
A 66-year-old man with Nitrates. coronary artery disease complains of headache after starting a medication for chronic stable angina. Associated with anxiety, Alcohol withdrawal. sweats, tactile hallucinations, agitation, and tremor. Extracranial infections. Sinusitis and dental abscess. Headache may be Temporomandibular joint disorder (TMJ). accompanied by clicking or grating sounds with chewing. Neck pain often occurs in Cervical spine disease (ie, cervicogenic headache). association with headache caused by this condition. A marathon runner presents Hypotonic hyponatremia. shortly after a race with acute headache, confusion, and vomiting. A painful red eye associated Primary angle-closure glaucoma. with headache, nausea, and vomiting. A 32-year-old man presents Extracranial carotid artery dissection. with sudden-onset neck pain and headache after visiting a chiropractor for a neck strain and is found to have ipsilateral ptosis and miosis. A 78-year-old woman Giant cell (or temporal) arteritis. complains of headache, scalp tenderness, and jaw discomfort with meals and is found to have an erythrocyte sedimentation rate of 80 mm/h.
What are the various ways in which medication can cause or contribute to headache? When does headache typically occur during the course of alcohol withdrawal?
What are the characteristics of the headache caused by sinusitis? What serious complication must be considered in a
Headache can occur as a side effect of appropriate medication use (eg, nitrates), as a result of medication overuse (eg, ibuprofen), or from medication withdrawal (eg, estrogens).
Minor withdrawal symptoms, including headache, typically occur within 8 hours of cessation of chronic alcohol consumption.27
Headache associated with sinusitis is typically periorbital with radiation to the ears, has a quality of pressure or dullness, is worse in the morning but improves over the course of the day, and is associated with nasal congestion.28 Any patient with a dental abscess who develops headache and a focal neurologic deficit should be evaluated with neuroimaging for a brain abscess as a result of contiguous spread of infection.
807
abscess, headache, and a
focal neurologic deficit? What are the characteristics of headache associated with temporomandibular joint disorder? What are the characteristics of cervicogenic headache? What is the mechanism of headache in patients with serum hypotonicity? What is the treatment for acute primary angle-closure glaucoma?
What is the typical location of the headache related to vertebral artery dissection? How common is headache in patients with giant cell arteritis?
Headache occurs in most patients with TMJ and is often the only manifestation. It is most commonly bilateral and can be located in the temporal, periorbital, or frontal regions. It is often precipitated by jaw movements, excessive talking, chewing gum, eating tough-textured food, and stress.29 Cervicogenic headache is 4 times more likely to occur in women. It is typically unilateral, starting at the back of the head before migrating to the front. It can be aggravated by certain neck positions.30 Serum hypotonicity leads to a shift of fluid from the extracellular the intracellular fluid compartment, with resultant cerebral edema and associated headache. A similar mechanism occurs when chronic hypertonicity is corrected too quickly. Acute primary angle-closure glaucoma is a medical emergency that requires prompt recognition and treatment to avoid blindness. Patients typically present with a painful red eye with blurred vision, headache, and nausea and vomiting. Topical and systemic pharmacologic agents (eg, topical timolol maleate and systemic acetazolamide) should be started immediately to lower intraocular pressure, followed by iridotomy. When recognized and treated promptly, most patients with acute primary angle-closure glaucoma recover without permanent vision damage.31 Vertebral artery dissection classically results in pain over the occipital region. When there is cerebral ischemia, headache may be accompanied by neurologic deficits in a posterior distribution pattern (eg, vision loss, nystagmus, cerebellar signs). The diagnosis is made with CT or magnetic resonance angiography.11 Giant cell arteritis typically affects patients older than 50 years. Headache is present in more than 90% of cases. It tends to be localized and progressive in nature, and may be accompanied by vision changes, systemic symptoms (eg, fever, weight loss, myalgias), and superficial temporal artery swelling and tenderness (see Figure 50-2). Elevation of the erythrocyte sedimentation rate (>50 mm/h) is characteristic. When giant cell arteritis is suspected, patients should immediately be started on high dose glucocorticoids (eg, 1 mg/kg of prednisone) followed by urgent biopsy of the temporal artery.11
808 Case Summary A previously healthy 50-year-old man with recent travel to British
Columbia presents with chronic fatigue, fever, headache, and tinnitus and is found to have an abnormal CSF profile with a ring-enhancing
brain lesion on neuroimaging. What is the most likely cause of headache in this patient? Fungal meningitis.
809
Bonus Questions
What features of this case ry Features of this case concerning for a secondary headache disorder include onset after 40 years of age, the presence of systemic symptoms and signs (fever What features of this casel In this case, fungal meningitisiis suggested by the indolent nature of the symptoms as well as the presence of the typical CSF profile of fungal meningitis Which organism is the most Cryptococcus gattii is the organism most likely to be involved in this case because it is endemic to British Columbia, Canada (where the patient had recently headache disorder? are suggestive of a seconda and weight loss), and the presence of neurologic deficits (tinnitus, papilledema). meningitis? are characteristic of funga (some atypical bacterial infect ons, such as Coxiella, can present with a similar CSF profile; see chapter 30, Meningitis). likely cause of fungal traveled) and can infect immunocompetent hosts. The markedly elevated opening pressure on lumbar puncture is also characteristic of cryptococcal meningitis in this case? meningitis. What cerebrospinal fluid CSF samples should be sent for India ink preparation, culture, and assays for cryptococcal antigen (including thelrecently developed lateral flow assay). studies can confirm the However, these studies are not 100% sensitive, particularly in cases involving Cryptococcus gattii. Repeat large-vo ume CSF samples can improve diagnosis of cryptococcal sensitivity. 32 meningitis? What is the most likely ing The brain lesion demonstrated by the neuroimaging in this case (see Figure 40-1, arrows) is most likely a cryptococcoma. cause of the ring-enhanc lesion on brain imaging in this case? What findings on MRI of the In addition to crytpococcomas, brain MRI in patients with cryptococcal meningitis may demonstrate dilated Virchow-Robin spaces, pseudocysts, cortical brain may be present in and lacunar infarcts, and hydrocephalus. 32 patients with cryptococcal meningitis? How is Cryptococcus Cryptococcusiis acquired via inhalation, where it can then disseminate. Sometimes dissemination occurs after a latent period, during which the fungus is Which 2 species ofe Cryptococcus neoformans is a ubiquitous fungus that typically infects immunocompromised hosts (eg, HIV-positive patients); Cryptococcus gattii is an hosts. acquired? contained w thin the lymph nodes of the lungs. 32 meningitis, and which Cryptococcus caus endemic fungus found in various regions of the world (eg, British Columbia, Canada) that can infect immunocompromised and immunocompetent demographic groups are affected by each? What is the natural history Without treatment, cryptococcal meningitis progresses and patients develop confusion, seizure, impaired consciousness, and eventually coma and of cryptococcal meningitis? death. 32 What is the treatment for Cryptococcal meningitisiis treated with antifungal therapy in 3 phases which include induction (2-6 weeks), consolidation (8tweeks), and maintenanceith cryptococcal meningitis? (≥1 year). The induction phase typically consists of the combination of liposomal amphotericin B plus flucytosine. Consolida ion and maintenance therapies consist of vary ng doses of fluconazole monotherapy. The recommended induction phase for immunocompetent patients (including those w infection) is longer (4-6 weeks). The presence of cerebral cryptococcomas may require prolonged therapy. Cryptococcus gattii 32 What is the prognosis ofin Outcomes are not appreciably different between HIV-positive patients infected with Cryptococcus gattii andtthose infected with Cryptococcus neoformans. In cryptococcal meningitis the industrialized world, the 10-week mortality in HIV-infected patients is as high as 25% despite treatmen . Factors associated with poor outcomes HIV-positive patients? include delirium at the time of presentation, older age, lower weight, and higher fungal burden (assessed via CSF colony-forming unit count). 32 What is the prognosis ofin Inlthe industrialized world, cryptococcal meningitis in HIV-negative patients is associated with a 90-day mortality rate of approximately 30%. Factorsd What therapeuticioption lho Cryptococcal meningitis with high CSF opening pressure is associatedlwith a greateriburden of symptoms (eg, headache, nausea, delirium). Therapeutic cryptococcal meningitis associated with poor outcomes include delirium at the time of presentation, the absence of headache, higher fungal burden, and lower CSF white bloo HIV-negative patients? ce l count. 32 have elevated intracrania pressure? exists for patients with lumbar punctures can be used to control high CSF pressure in these patients, improv ng symptoms and other outcomes, including mortality. One 2 cryptococcal men ngitis w approach is to reduce the opening pressure by 50% (when extremely e evated) or to a normal pressure of <20 cm H O. 33,34
810 Key Points
There isia high lifetime prevalence of headache in the general populat on, and it is one of the most common presenting complaints in the primary care setting. Headache disorders can be primary or secondary.
The most common primary headache disorders include tension headache, migraine headache, and cluster headache. In general, primary headache disorders are more common than
secondary disorders, but the risk of a secondary disorder increases Clues to the presence of secondary headache include sudden or in certain populations (eg, immunocompromised patients). new-onset headache at an older age (>40 years), change in the quality of chronic headache, systemic symptoms or illness (eg,
fever), neurologic symptoms or signs (eg, seizure), headaches that wake patients from sleep, and headaches that are worsened with Valsalva.
Secondary headache disorders can be caused by intracranial or Intracranial causes of secondary headache can be separated into cerebrospinal fluid, and other. extracranial processes. the following subcategories: vascular, infectious, tumor,
811
References 1. Bigal ME, Bordini CA, Speciali JG. Etiology and distribution of headaches in two Brazilian primary care units. Headache. 2000;40(3):241-247. Hale N, Paauw DS. Diagnosis and treatment of headache in the ambulatory care
-
setting: a review of classic presentations and new considerations in diagnosis and management. Med Clin North Am. 2014;98(3):505-527.
-
Venkatesan A. Case 13: a man with progressive headache and confusion. MedGenMed. 2006;8(3):19.
-
Estemalik E, Tepper S. Preventive treatment in migraine and the new US guidelines. Neuropsychiatr Dis Treat. 2013;9:709-720.
-
Gilmore B, Michael M. Treatment of acute migraine headache. Am Fam Physician. 2011;83(3):271-280.
-
Newman LC. Trigeminal autonomic cephalalgias. Continuum (Minneap Minn). 2015;21(4 Headache):1041-1057.
-
Hajj-Ali RA, Calabrese LH. Primary angiitis of the central nervous system. Autoimmun Rev. 2013;12(4):463-466.
-
Scott RM, Smith ER. Moyamoya disease and moyamoya syndrome. N Engl J Med. 2009;360(12):1226-1237.
-
Spierings EL. Acute and chronic hypertensive headache and hypertensive encephalopathy. Cephalalgia. 2002;22(4):313-316.
-
Moser M, Wish H, Friedman AP. Headache and hypertension. JAMA. 1962;180:301- 306.
-
Dodick D. Headache as a symptom of ominous disease. What are the warning signals? Postgrad Med. 1997;101(5):46-50, 5-6, 62-4.
-
Raps EC, Rogers JD, Galetta SL, et al. The clinical spectrum of unruptured intracranial aneurysms. Arch Neurol. 1993;50(3):265-268. . Sattar A, Manousakis G, Jensen MB. Systematic review of reversible cerebral
13vasoconstriction syndrome. Expert Rev Cardiovasc Ther. 2010;8(10):1417-1421. Walker HK, Hall WD, Hurst JW, eds. Clinical Methods: The History, Physical, and Laboratory Examinations. 3rd ed. Boston: Butterworths;1990. . Tunkel AR, Glaser CA, Bloch KC, et al. The management of encephal tis: clinical
15practice guidelines by the Infectious Diseases Society of America. CliniInfect Dis. 2008;47(3):303-327.
- Pittenger B, Young JW, Mansoor AM. Subretinal abscess. BMJ Case Rep. 2017;2017. 17toxoplasmosis in a cohort of human immunodeficiency virus-infected patients: 1988- . Belanger F, Derouin F, Grangeot-Keros L, Meyer L. Incidence and risk factors of
-
HEMOCO and SEROCO Study Groups. Clin Infect Dis. 1999;28(3):575-581.
-
DeAngelis LM. Brain tumors. N Engl J Med. 2001;344(2):114-123.
-
Forsyth PA, Posner JB. Headaches in patients with brain tumors: a study of 111 patients. Neurology. 1993;43(9):1678-1683.
-
Rivkin M, KanoffiRB. Metastatic brain tumors: current therapeutic options and historical perspect ve. J Am Osteopath Assoc. 2013;113(5):418-423.
- Owonikoko TK, Arbiser.J, Zelnak A, et al. Current approaches to the treatment of metastatic brain tumours Nat Rev Clin Oncol. 2014;11(4):203-222.
- Ohgaki H. Epidemiology of brain tumors. Methods Mol Biol. 2009;472:323-342.
812
-
Pople IK. Hydrocephalus and shunts: what the neurologist should know. J Neurol Neurosurg Psychiatry. 2002;73(suppl 1):i17-i22.
-
Corbett JJ,tSavino PJ, Thompson HS, et(al. Visual loss in pseudotumor cerebri. Follow- up of 57 pa ients from five to 41 years and a profile of 14 patients with permanent severe visual loss. Arch Neurol. 1982;39 8):461-474.
- Zakrzewska JM, Linskey ME. Trigeminal neuralgia. BMJ Clin Evid. 2014;2014.
-
Hoffman JM, Lucas S, Dikmen S, et al.:Natural history of headache after traumatic 27. Schuckit MA. Recognition and management of withdrawal delirium (delirium brain injury. J Neurotrauma. 2011;28(9) 1719-1725. tremens). N Engl J Med. 2014;371(22):2109-2113.
-
Tarabichi)M. Characteristics of sinus-related pain. Otolaryngol Head Neck Surg. 2000;122(6 :842-847.
-
Lupoli TA, Lockey RF. Temporomandibular dysfunction: an often overlooked cause of chronic headaches. Ann Allergy Asthma Immunol. 2007;99(4):314-318.
-
Page P. Cervicogenic headaches: an evidence-led approach to clinical management. Int J Sports Phys Ther. 2011;6(3):254-266.
-
Weinreb RN, Aung T, Medeiros FA. The pathophysiology and treatment of glaucoma: a review. JAMA. 2014;311(18):1901-1911.
-
Williamson PR, Jarvis JN, Panackal AA, et al. Cryptococcal meningitis: epidemiology, immunology, diagnosis and therapy. Nat Rev Neurol. 2017;13(1):13-24. . Graybill JR, Sobel J, Saag M, et al. Diagnosis and management of Increased
33intracranial pressure in patients with AIDS and cryptococcal meningitis. The NIAID Mycoses Study Group and AIDS Cooperative Treatment Groups. Clin Infect Dis.
2000;30(1):47-54. 34management of cryptococcal disease: 2010.update by the Infectious Diseases Society of . Perfect JR, Dismukes WE, Dromer F, et al Clinical practice guidelines for the America. Clin Infect Dis. 2010;50(3):291-322.
813
CHAPTER 41
814
Polyneuropathy
815 Case: A 42-year-old woman with a painful tongue A 42-year-old woman with a history of hypertension, obesity, and
gastric bypass surgery complicated by chronic diarrhea is evaluated in weeks. She also describeslprogressively worsening numbness and period of time. She describes difficulty with balance and walking. The stopped taking all medications 8 months ago because of financial the clinic for a painful tongue and numbness of the lower extremities. The tongue has been swo len, red, and painful over the past few “pins and needles” sensations in the hands and feet over the same patient underwent gastric bypass surgery at 38 years of age. She reasons. She does not drink alcohol. There is erythema and scaling of the oral commissures, and the
tongue is beefy andtred (Figure 41-1). There are sharply.marginated patches of scaly ery hema around the anogenital region There is symmetric weakness of the hip and knee flexors. Patellar tendon reflexes are brisk, however Achilles tendon reflexes are absent.
Stimulation of the plantar aspect of the foot results in dorsiflexion of deficits in light touch, vibratory sensation, and proprioception in all 4 sensations. Romberg test is positive. There is frank ataxia on gait the hallux (ie, positive Babinski sign). There are symmetric distal extremities, and there are less severe deficits in pain and temperature examination.
FIGURE 41-1 (Courtesy of OHSU Chief Residents, 2012-2013.)
What is the most likely cause of polyneuropathy in this patient?
What is Neuropathy describes injury to any part of the central or peripheral nervous systems (Figure 41-2). neuropathy?
816
FIGURE 41-2 The central and peripheral nervous systems.
What is peripheral The peripheral nervous system is composed of the cranial nerves (excluding the optic nerve), the spinal nerve roots neuropathy? (ventral and dorsal), the dorsal root ganglia, the peripheral nerve trunks and associated terminal branches, and the peripheral autonomic nervous system. Peripheral neuropathy refers to injury or dysfunction in any part of the peripheral nervous system, encompassing conditions such as radiculopathy, plexopathy, mononeuropathy, and polyneuropathy. The somatic (ie, motor, sensory) and autonomic nervous systems may be involved. Clinical manifestations may include pain, impaired sensation and strength, and autonomic symptoms (eg, abnormal sweating). The term peripheral neuropathy is often incorrectly used as a synonym for polyneuropathy.1–3 What is Radiculopathy is a type of peripheral neuropathy in which injury occurs at the level of the nerve root, where the nerve radiculopathy? exits the spinal cord. Clinical manifestations involve the corresponding dermatome and myotome. Sensory symptoms (eg, numbness, tingling, pain) often start in the back or neck and radiate to an extremity in a dermatomal distribution.4 What is Plexopathy is a type of peripheral neuropathy involving groups of nerves called plexuses, usually related to trauma plexopathy? or entrapment (ie, compression), resulting in symptoms and signs in the distribution of the plexus. For example, brachial plexopathy results in disturbances in strength, sensation, and reflexes along the C5-T1 distribution.5 What is Mononeuropathy is a type of peripheral neuropathy involving a single peripheral nerve, usually related to trauma or mononeuropathy? entrapment. Clinical manifestations are typically acute to subacute in onset and correlate with an individual nerve distribution. Carpal tunnel syndrome is an example of mononeuropathy.4 What is Mononeuritis multiplex (ie, multiple mononeuropathy or mononeuropathy multiplex) is a type of peripheral mononeuritis neuropathy that involves at least 2 noncontiguous peripheral nerves. It is commonly associated with vasculitis, where multiplex? inflammation and thrombosis of the vasa nervorum results in ischemic injury to the nerve. Other causes of mononeuritis multiplex include diabetes, leprosy, sarcoidosis, and hereditary neuropathy with liability to pressure palsy.3,6 What is Polyneuropathy is a type of peripheral neuropathy that is generalized, affecting many peripheral nerves. Clinical polyneuropathy? manifestations typically occur in symmetric nerve distributions, usually in a length-dependent fashion (ie, disturbances begin at the terminal ends of the longest nerves), and with an early predominance of sensory symptoms (weakness is a late finding). Although mononeuritis multiplex is technically a form of polyneuropathy and can mimic a generalized condition (particularly when it is advanced), it is considered separate by convention.4 What are the 2 Neuropathy can be associated with axonal or demyelinating patterns of injury (Figure 41-3). Axonal neuropathy main involves degeneration and loss of axons. Demyelinating neuropathy involves degeneration of the myelin surrounding pathophysiologic axons. Causes of neuropathy can be associated with patterns of axonal or demyelinating injury, which can guide the and electrophysiologic differential diagnosis.1 patterns of nerve injury in patients with polyneuropathy?
FIGURE 41-3 Axonal degeneration and demyelinating disease of peripheral nerve fibers. A, The normal axon surrounded
What are the symptoms of polyneuropathy? What parts of the neurologic examination are important in the evaluation of
by myelin, which provides an insulating sheath that allows for rapid saltatory conduction of action potentials. B, Axonal degeneration (“dying-back” phenomenon). C, Demyelination, with loss of the myelin sheath. (From Krishnan AV, Pussell BA, Kiernan MC. Neuromuscular disease in the dialysis patient: an update for the nephrologist. Semin Dial. 2009;22:267-278.)
Symptoms of polyneuropathy may include, in varying combinations, numbness or loss of sensation (hypoesthesia), “pins and needles” or tingling sensations (paresthesia), pain or burning (dysesthesia), early satiety or other autonomic disturbances, and weakness (typically a late finding).4 Physical examination is essential in patients with polyneuropathy. The cranial nerve examination is important for identifying proximal involvement. Motor examination should assess for strength, muscle bulk, tone, and the presence of fasciculations. Reflexes should be evaluated. Sensory examination should separately evaluate the large fibers (ie, vibration and joint position) and small fibers (ie, pain [assessed by pinprick testing] and temperature). The pattern of motor and sensory involvement should be noted (ie, symmetric or asymmetric, distal or proximal, focal or nonfocal).
817
patients with polyneuropathy? suspected What are the physical findings of polyneuropathy?
Other useful tests include the Romberg test and evaluation of gait.3,7
Physical findings of polyneuropathy depend on the predominant type of nerve fibers involved (ie, motor or sensory), but may include, in varying combinations, hypoesthesia, hyperesthesia, allodynia, sensory ataxia, hyporeflexia, generalized weakness, and autonomic disturbances (eg, orthostatic hypotension).8
Why is it helpful to Different causes of polyneuropathy often affect nerve fibers in specific patterns. Physical testing can identify determine the size of fibers the fibers involved, thus narrowing the differential diagnosis. Large fiber neuropathies manifest with involved in patients with combinations of disturbances in strength, vibratory sense, and proprioception; whereas small fiber polyneuropathy? neuropathies manifest with combinations of disturbances in pain and temperature sensations, and autonomic function. Reflexes are generally diminished or absent in patients with large fiber neuropathies, but remain intact in patients with small fiber neuropathies.3 In addition to Although some types of polyneuropathies predominantly involve motor fibers, a pure motor disorder (ie, polyneuropathy, what weakness without sensory or autonomic disturbances) should prompt consideration of motor neuron other lower motor neuron diseases (eg, amyotrophic lateral sclerosis), neuromuscular junction disorders (eg, myasthenia gravis), and conditions should be myopathies (eg, polymyositis). considered in patients who present with pure motor disturbance? What is the role of Electrodiagnostic studies (ie, nerve conduction study [NCS] and electromyography [EMG]) are useful for electrodiagnostic testing in identifying disorders involving large myelinated fibers but not small fibers. The combination of NCS and the workup of EMG can localize the lesion, determine the pattern of injury (ie, axonal or demyelinating), determine the polyneuropathy? fiber type involved (ie, motor, sensory, or both), and provide a sense of severity and time course of the disease (ie, acute or chronic).9,10 What studies are available Autonomic function can be assessed via the following methods: (1) measuring the heart rate response to to test for autonomic respiration, Valsalva maneuver, or tilting (ie, tilt table testing); (2) measuring the blood pressure response dysfunction? to hand grip, Valsalva maneuver, or tilting; and (3) measuring the sympathetic skin response (ie, sudomotor testing).10 What supplementary Genetic studies, cerebrospinal fluid (CSF) examination, skin biopsy, and nerve biopsy can each play a role in studies can be helpful in the evaluation of polyneuropathy in select patients. select patients with polyneuropathy? How common is Polyneuropathy affects about 4% of the general population in the industrialized world, and is more common polyneuropathy? in patients older than 75 years.4 What are the 4 general Polyneuropathy can be metabolic, toxic, inflammatory, or hereditary. categories of polyneuropathy?
How often is a diagnosis made in patients presenting with polyneuropathy?
The combination of history, physical examination, laboratory testing (eg, serum protein electrophoresis [SPEP]), and ancillary testing (eg, electrodiagnostic tests) reveals a diagnosis in up to 80% of cases of polyneuropathy. A diagnosis is not made in the remainder of cases despite a thorough workup. Idiopathic polyneuropathy typically progresses slowly and does not often result in severe physical disability.10
818
Metabolic Causes of Polyneuropathy
What pathophysiologic or electrophysiologic pattern of nerve injury is most commonly associated with metabolic causes of polyneuropathy?
Metabolic causes of polyneuropathy are most commonly associated with axonal injury, but demyelinating injury can predominate in some conditions.3
What are the metabolic causes of polyneuropathy?
The single most common cause of polyneuropathy in the industrialized world. Weight gain, bradycardia, and Queen Anne’s sign (thinning of the lateral third of the eyebrows). In patients with this condition, progressive polyneuropathy is an indication to initiate dialysis, which is often successful in halting further neuropathic progression. A patient with celiac sprue presents with paresthesias of the feet. Spider angiomas, palmar erythema, and caput medusae. Frontal bossing, protruding jaw (Figure 41-4), and hyperglycemia.
Diabetes mellitus.
Hypothyroidism.
Chronic kidney disease (CKD).11
Nutritional deficiency. Chronic liver disease. Acromegaly.
FIGURE 41-4 A, Typical coarse facial features of acromegaly. The nose, eyelids, and ears are enlarged and thickened. The supraorbital ridges are prominent. The lower lip is thick and projecting. The bones of the lower jaw are enlarged, leading to prognathism. B, Skull of a patient with acromegaly, demonstrating prognathism and prominent supraorbital ridges. (From Osborne OT. Acromegaly. In: Buck AH, ed. A Reference Handbook of the Medical Sciences. Vol 1. New York: William Wood and Co.; 1900.)
Which screening tests can be performed in the clinic to evaluate for polyneuropathy in patients with diabetes?
What are the clinical characteristics of polyneuropathy related to hypothyroidism? How common is polyneuropathy in patients with chronic kidney disease?
Which nutritional deficiencies are associated with polyneuropathy?
Acquired copper deficiency can develop in association with exposure to high levels of which other micronutrient? What are the clinical characteristics of polyneuropathy
Diabetes mellitus accounts for up to one-half of cases of polyneuropathy in the industrialized world. In diabetic patients, impairment in vibratory perception using a 128-Hz tuning fork is associated with a positive likelihood ratio of polyneuropathy of up to 35; impairment in pressure sensation with a 5.07 monofilament is associated with a positive likelihood ratio of up to 15. Prediabetes (eg, impaired glucose tolerance) can also cause polyneuropathy.4 Polyneuropathy related to hypothyroidism is common, although typically mild. Electrodiagnostic testing most often reveals a sensory neuropathy. Paresthesia is the most commonly reported symptom.12 Polyneuropathy is present in the majority of patients with CKD who are dialysis-dependent. The high prevalence in this population is likely a reflection of the coexistence of diabetes and CKD. Patients with the combination of CKD and diabetes tend to have more severe polyneuropathy than those with CKD alone.11 Polyneuropathy can be caused by deficiencies of vitamin B12 (cobalamin), vitamin B1 (thiamine), vitamin B6 (pyridoxine), vitamin E, and copper.13
High zinc exposure (eg, related to nutritional supplementation or exposure to some denture pastes) is associated with copper deficiency.10
Most patients with polyneuropathy related to chronic liver disease are asymptomatic. Sensory neuropathy is the most commonly identified type, the pattern of nerve injury is usually axonal, and
819
Which type of peripheral neuropathy is most common in patients with acromegaly?
Polyneuropathy is an uncommon complication of acromegaly. More typically, these patients develop mononeuropathies (eg, carpal tunnel syndrome) related to entrapment by bone and soft tissue hypertrophy.15
820
Toxic Causes of Polyneuropathy
What pathophysiologic or electrophysiologic pattern of nerve injury is most commonly associated with toxic causes of polyneuropathy?
Toxic causes of polyneuropathy are most commonly associated with axonal injury, but demyelinating injury can predominate in some conditions.3
What are the toxic causes of polyneuropathy?
The second most common cause of polyneuropathy in the industrialized world, usually occurring after decades of daily exposure. Iatrogenic. A patient develops polyneuropathy after spending the past several weeks in an intensive care unit with acute respiratory distress syndrome. A 56-year-old man who works in a factory producing batteries for vehicles presents with bilateral lower extremity paresthesias and is found to have microcytic anemia and basophilic stippling of the erythrocytes on peripheral blood smear (Figure 41-5).
Alcohol.4
Medication. Critical illness polyneuropathy.
Lead toxicity.
FIGURE 41-5 Microcytic red blood cells with coarse basophilic stippling
A 21-year-old college student presents with polyneuropathy after starting a job as a technician in a biochemistry laboratory where he uses gel electrophoresis to study proteins. A 54-year-old farmer presents with a motor-predominant polyneuropathy 2 weeks after experiencing an episode of excessive eye tearing, excessive sweating, diarrhea, and vomiting. This vitamin is unique in that it can be associated with polyneuropathy when it is either deficient or present in excess.
Which metabolic causes of polyneuropathy are often present in alcoholic patients?
What medications are associated with polyneuropathy?
What risk factors are associated with critical illness polyneuropathy? Which heavy metals are associated with polyneuropathy? What is the prognosis of polyneuropathy related to acrylamide toxicity? What is the prognosis of polyneuropathy related to organophosphate toxicity?
(arrows) in a patient with lead poisoning. (From: Pereira I, George TI, Arber DA. Atlas of Peripheral Blood: The Primary Diagnostic Tool. Philadelphia, PA: Wolters Kluwer Health; 2012.)
Acrylamide toxicity.
Organophosphate poisoning.
Vitamin B6.
Long-term excessive alcohol use can cause polyneuropathy via direct neurotoxicity. These patients often additionally suffer from nutritional deficiencies and chronic liver disease, both of which may contribute to the polyneuropathy of alcoholism.10 Numerous medications are associated with polyneuropathy; including chemotherapeutic agents (eg, vincristine, paclitaxel), antibiotics (eg, metronidazole, nitrofurantoin, isoniazid, dapsone), antiretroviral agents (eg, zalcitabine, didanosine), cardiac medications (eg, amiodarone), colchicine, phenytoin, cimetidine, lithium, and disulfiram.3 Risk factors for critical illness polyneuropathy include sepsis, systemic inflammatory response syndrome, and multiple organ failure.16 Exposure to lead, thallium, arsenic, or mercury can result in polyneuropathy.17
Nearly complete reversal of polyneuropathy occurs when exposure to acrylamide is removed.17 Mild cases of polyneuropathy related to organophosphate toxicity are associated with good recovery; in cases where myelopathy is also present, recovery is generally poor.17
821
What dose of vitamin B6 is associated with polyneuropathy?
What additional occupational toxins can lead to polyneuropathy?
The polyneuropathy related to vitamin B6 toxicity usually occurs when doses are greater than 2 g/d, but it has also been reported when lower doses are consumed over long periods of time. 17 Polyneuropathy can occur as a result of exposure to allyl chloride, carbon disulfide, dimethylaminopropionitrile, ethylene oxide, and hexacarbons (eg, n- hexane).17
822
Inflammatory Causes of Polyneuropathy
What pathophysiologic or electrophysiologic pattern of nerve Inflammatory causes of polyneuropathy are most commonly injury is most commonly associated with inflammatory causes of associated with demyelinating injury, but axonal injury can polyneuropathy? predominate in some conditions.3 What are the typical cerebrospinal fluid findings in patients with Inflammatory causes of polyneuropathy are often associated with polyneuropathy related to inflammatory conditions? elevated CSF protein concentration with or without pleocytosis.7 What are the 2 subcategories of inflammatory polyneuropathy? Inflammatory polyneuropathy can be infectious or noninfectious.
823
Infectious Causes of Inflammatory Polyneuropathy
What are the infectious causes of inflammatory polyneuropathy?
Polyneuropathy can be associated with this chronic viral infection or its treatment. A patient visits a clinic in Boston for evaluation of symmetric, distal paresthesias. He recalls a target- shaped rash on his leg approximately 8 months prior. Development of cranial and upper extremity neuropathy 4 weeks after an upper respiratory tract infection characterized by ulcerative oropharyngeal mucosa and the presence of a pseudomembrane. One of the most common causes of peripheral neuropathy worldwide, this infection most typically presents with mononeuropathy or mononeuritis multiplex, particularly of the upper extremities; patients may also have hypopigmented skin lesions with hypoesthesia.
How common is polyneuropathy in patients with HIV infection?
What is the prognosis of polyneuropathy associated with Lyme disease? How common is polyneuropathy in patients with diphtheria?
Are anesthetic skin lesions universally present in patients with neuropathy related to leprosy?
Human immunodeficiency virus (HIV). Lyme disease.
Diphtheria.
Leprosy.18
Polyneuropathy is estimated to affect up to one-third of patients with HIV. It commonly develops within months of HIV transmission. Most patients complain of symmetric distal numbness, burning pain, and paresthesias. Examination reveals Decreased or absent Achilles tendon reflexes, and deficits in vibratory and temperature sensations in a stocking-glove distribution.19 Most cases of polyneuropathy associated with Lyme disease resolve or improve after treatment of the underlying infection.20 Polyneuropathy develops in the majority of patients with severe respiratory infection from diphtheria and in a minority but significant proportion of patients with asymptomatic infection. The first indication of neuropathy related to diphtheria is paralysis of the soft palate and posterior pharyngeal wall; the descendent nature of the polyneuropathy can help distinguish it from Guillain- Barré syndrome (GBS).21 Classic leprosy neuropathy presents with anesthetic skin lesions, but pure neuritic leprosy presents with neuropathy in the absence of cutanous manifestations. Nerve biopsy is the gold standard for diagnosis in such cases.18
824
Noninfectious Causes of Inflammatory Polyneuropathy
What are the noninfectious causes of inflammatory polyneuropathy?
Two weeks after a diarrheal illness, a 36- Guillain-Barré syndrome. year-old woman presents with ascending paralysis and is found to have elevated CSF protein concentration without pleocytosis (ie, albuminocytologic dissociation). A heterogeneous group of disorders that Acquired amyloidosis.22 result in the deposition of misassembled fibril proteins that alter the structure and function of normal tissues, most commonly within the kidneys, liver, and heart. Diagnosed using SPEP, and sometimes Paraproteinemia. associated with a low serum anion gap. A 63-year-old man with an extensive Paraneoplastic syndrome. smoking history presents with idiopathic polyneuropathy and is found to have positive serum anti-Hu antibodies. Malar rash, pancytopenia, and Systemic lupus erythematosus (SLE). polyarticular inflammatory arthritis. A patient with palpable purpura, Vasculitis (eg, cryoglobulinemia). hematuria with dysmorphic erythrocytes (see Figure 34-4), and foot drop. A 44-year-old woman presents with Chronic inflammatory demyelinating polyneuropathy (CIDP). progressive symmetric polyneuropathy and hyporeflexia over the course of 10 weeks and is found to have slowed nerve conduction on electrodiagnostic testing and albuminocytologic dissociation on evaluation of CSF. A granulomatous condition. Sarcoidosis.
What is the treatment for Guillain-Barré syndrome?
What are the characteristics of polyneuropathy related to acquired amyloidosis?
What conditions are associated with paraproteinemic polyneuropathy?
When does paraneoplastic polyneuropathy usually occur during the course of malignancy? Which connective tissue diseases are associated with polyneuropathy? What types of peripheral neuropathy
Treatment with plasma exchange or intravenous immunoglobulin (IVIG) hastens recovery in patients with Guillain-Barré syndrome, particularly when initiated within 2 weeks of disease onset. Despite treatment, mortality is around 5% and long-term severe disability occurs in approximately 20% of patients.23 Peripheral neuropathy most commonly occurs in the setting of immunoglobulin light chain (AL) amyloidosis. It is a length-dependent sensorimotor polyneuropathy, but involves the autonomic nervous system in most cases, causing patients to present with symptoms such as nausea, vomiting, early satiety, bloating, constipation, diarrhea, postural light-headedness, and erectile dysfunction. Peripheral neuropathy is rare in inflammatory (AA) amyloidosis.22 Paraproteinemic polyneuropathy can occur with monoclonal gammopathy of unknown significance (most common), multiple myeloma, AL amyloidosis, POEMS syndrome (Polyneuropathy, Organomegaly, Endocrinopathy, Monoclonal gammopathy, and Skin changes), Waldenström macroglobulinemia, cryoglobulinemia (types I and II), and lymphoproliferative disorders (eg, chronic lymphocytic leukemia). 24
In patients with paraneoplastic polyneuropathy, the polyneuropathy often precedes the diagnosis of the underlying malignancy, which can sometimes be discovered with investigative imaging studies, including whole body positron emission tomography (PET).25 Polyneuropathy can occur in patients with SLE, rheumatoid arthritis, and Sjögren’s syndrome.26 Patients with vasculitis most frequently develop polyneuropathies and mononeuropathies,
825
What treatments are associated with improved outcomes in patients with chronic inflammatory demyelinating polyneuropathy?
How common is peripheral neuropathy in patients with sarcoidosis?
Systemic glucocorticoids, IVIG, and plasma exchange are associated with improved outcomes in patients with CIDP. Treatment choice depends on variables related to the host and features of the disease (eg, patients with pure motor CIDP should not be treated with glucocorticoids, as there is risk of deterioration with this modality). Sometimes switching or combining modalities may be necessary.27 Sarcoidosis involves the nervous system in approximately 5% of cases. Cranial neuropathy (especially involving the facial nerve) is the most common manifestation, affecting up to one-half of patients. Other peripheral neuropathies, including mononeuritis multiplex, radiculopathy, and polyneuroapthy, are less common. In some cases, an acute GBS-like polyneuropathy can occur. However, the CSF profile shows elevated protein and pleocytosis, distinguishing it from GBS, which typically presents with albuminocytologic dissociation.28
826
Hereditary Causes of Polyneuropathy
What pathophysiologic or electrophysiologic pattern of nerve injury is most commonly associated with hereditary causes of polyneuropathy?
Hereditary causes of polyneuropathy are most commonly associated with demyelinating injury, but axonal injury can predominate in some conditions.3
What are the hereditary causes of polyneuropathy?
The most common hereditary cause of polyneuropathy, ultimately diagnosed in one-fifth of patients presenting to neuromuscular clinics with chronic peripheral neuropathy. A 39-year-old Portuguese man presents with numbness in the feet, early satiety, and light-headedness on standing and is subsequently diagnosed with a fat pad biopsy. A 25-year-old woman presents with recurrent episodes of bilateral lower extremity weakness and numbness that usually occur after periods of prolonged sitting and last for weeks at a time. Intermittent acute attacks of diffuse abdominal pain, paresthesias, and red urine (see Figure 42- 4). A group of hereditary disorders, most of which present at birth with a variety of sensory and autonomic abnormalities. Disorders of maternal lineage.
What foot deformities are common in patients with Charcot-Marie-Tooth disease?
Charcot-Marie-Tooth (also known as hereditary motor and sensory neuropathy) type 1.29,30
Familial amyloidosis.
Hereditary neuropathy with liability to pressure palsy (HNPP).
Acute porphyria.
Hereditary sensory and autonomic neuropathies (HSAN). Some of the HSANs are misnomers, as motor neuropathy occurs with a high frequency in many of these disorders.31 Mitochondrial disorders.
Pes cavus (high arched feet) and hammertoes (the proximal interphalangeal joint of the toe is bent upward) are common in patients with Charcot-Marie-Tooth disease (Figure 41-6).30
FIGURE 41-6 Pes cavus with hammertoes in a patient with Charcot-Marie-Tooth
What patterns of peripheral neuropathy are characteristic of familial amyloidosis?
What is the prognosis of hereditary neuropathy with liability to pressure palsy?
What are the characteristics of the peripheral neuropathy associated with porphyria?
Which of the hereditary sensory and autonomic neuropathies can present in adulthood?
disease. (From Louis ED, Mayer SA, Rowland LP. Merritt’s Neurology. 13th ed. Philadelphia, PA: Wolters Kluwer; 2016.)
Patients with familial amyloidosis can develop mononeuropathy, sensorimotor polyneuropathy, autonomic neuropathy, or a combination of the 3. The most common and earliest site of involvement is the median nerve at the wrist, which presents like carpal tunnel syndrome.22 Patient with HNPP usually recover completely from acute neuropathies, but it can sometimes take several months. Chronic (usually mild) symptoms may persist, and the phenotype of patients with residual symptoms can resemble that of CMT. Life expectancy is normal.29 In patients with porphyria, motor and autonomic disturbances predominate, with an axonal pattern of nerve injury. Motor neuropathy can be asymmetric, affecting proximal (including facial) and distal muscles. Attacks can be precipitated by hormonal factors, medications, nutritional status, and alcohol consumption.32 HSAN type 1 often presents in early adulthood (median age of 20 years with a range of 15-55 years) with sensory symptoms, followed by weakness, ulcers, pain, and
827
balance difficulties.31 How common is peripheral neuropathy in Peripheral neuropathy is present in almost one-half of patients with mitochondrial patients with mitochondrial disorders? disorders and can be the predominant feature, often presenting in adulthood (eg, mitochondrial neuroGastrointestinal encephalopathy).33 What are some general hereditary disorders Spinocerebellar ataxia, Tangier disease, abetalipoproteinemia, Chediak-Higashi associated with polyneuropathy?disorders syndrome, and lysosomal storage diseases can be associated with polyneuropathy. Some hereditary associated with peripheral neuropathy that manifest predominantly in infancy or childhood (eg, giant axonal neuropathy) have been omitted to limit the scope of this chapter.
828 Case Summary prA 42-year-old woman with a history of gastric bypass surgery esents with skin changes, glossitis, and evidence of myeloneuropathy.
What is the most likely cause of polyneuropathy in this patient? Vitamin B12 deficiency.
829
Bonus Questions
Why is the Bariatric surgery can lead to malabsorption of micronutrients such as vitamin B , folate, iron, zinc, and copper. The patient in this case stopped taking supplemental 12 patient in this case at risk for micronutrients, which is usually necessary for patients after bariatric surgery, putting her at Increased risk for deficiencies. vitamin B 1 deficiency?2 significance ofgs including vitamin Bh. The scaly erythematous rash around the anogenital area probably represents acquired acrodermatitis enteropathica related totzinc deficiency. s, What is the of The Romberg test evaluates for deficits in proprioception (large fibers). The patient is asked to stand with the feet together. To maintain balance in this position, at least Romberg test in on the vestibular system, which is not enough to maintain balance. What is the The scaling and eryt ema of the oral commissures described in this case is suggestive of angular cheilitis, which is associated with a number of nutri ional deficiencie 12 the skin findin in this case? significance 2 of the following senses are necessary: normal vision, normal vestibular function, and normal proprioception. In a patient with a disturbance in proprioception, the positive balance can be maintained by normal vestibular function and vision. However, when the eyes are closed, a loss of balance occurs as the patient becomes solely reliant this case? What is the The brisk patellar tendon reflexes, proximal muscle weakness, positive Babinski sign, and impairment in vibratory/proprioceptive sense greater han the impairmen significance of n in pain/temperature sense are suggestive of spinal cord (corticospinal tract and dorsal column) involvement. The absent Achilles tendon reflexestindicate peripheralt of brisk patellar 12 the combinatio nerve involvement. The combination of both spinal cord and peripheral nerve involvement is known as myeloneuropathy. Patients with vitamin B deficiency often tendon reflexes present with myeloneuropathy, which can be a clue to the diagnosis.13,34 Achillesitendon and absent reflexes n this case? What would be The polyneuropathy of vitamin B12 deficiency is axonal in most cases.34 electrodiagnostic the expected pattern of nerve injury in this case? How common is Peripheral neuropathy is seen in one-quarter of patients with vitamin B12 deficiency.34 peripheral neuropathy in
patients with vitamin B 1 deficiency?2 What are the Foods rich in vitamin B12 include animal proteins, such as meats and eggs.34 major dietary sources of12? vitamin B What is the The recommended daily allowance of vitamin B is 2.4 µg for men and nonpregnant women, 2.6 µg for pregnant women, 2.8 µg for lactating women, and 1.5 to 2 µg 12 recommended daily intake of for children up to 18 years of age.34 vitamin B ? 12 In the The average non-vegetarian diet in the industrialized world contains around 5 to 8 µg of vitamin B per day; a vegetarian diet contains ≤0.5 µg of vitamin B per 12 12 industrialized world, how day, putting these individuals at risk for deficiency.34 much vitamin 12 B is found in the average diet? What is the The low pH of the stomach allows for the release of vitamin B from its coenzyme form via the actions of pepsin. Vitamin B then binds to haptocorrin (R-protein), and the complex travels to the second part of the duodenum, where pancreatic proteases free vitamin B . It then binds intrinsic factor (IF), and the complex is 12 12 12 basic physiology of vitamin B absorption? 12 absorbed in the ileum.34 What is the most The most common cause of vitamin B in the industrialized world is pernicious anemia, where immune-mediated atrophy of the gastric parietal cells results in 12 common cause of 12 vitamin B impaired secretion of IF and hydrochloric acid, which are both integral to vitamin B12 absorption.34 deficiency in the industrialized world? What Vitamin B12 deficiency is associated with macrocytosis, with or without anemia. can be revealed abnormalities on a complete blood count in the setting of 12 vitamin B deficiency? What No single biochemical laboratory test should be considered diagnostic of vitamin B deficiency in the absence of a compatible clinical syndrome or other corroborating biochemical tests. Serum vitamin B concentration is useful, particularly in patients with compatible hematologic or neurologic manifestations, where it is below normal in virtually all cases. However, a number of conditions (eg, multiple myeloma) can influence serum concentrations of vitamin B , leading to falsely low or high values. In patients with borderline vitamin B levels (between 200 and 300 pg/mL) or when clinical suspicion remains high despite vitamin B levels at 12 biochemical laboratory tests are useful in the diagnosis of 12 12 12 12 vitamin B12 the lower end of normal (<350 pg/mL), testing serum concentrationsiof methylmalonic acid and homocysteine may be useful; both metabolites are elevated in vitamin 12 12 B deficiency (normal levels suggest the absence of vitamin B def ciency). A number of variables (eg, renal insufficiency, age) can falsely elevate the serum 12 deficiency? concentrations of methylmalonic acid and homocysteine. A reduction in serum concentration of these metabolites after vitamin B supplementation can provide confirmation of true vitamin B12 deficiency.8,34,35 What is the Vitamin B should be adminstered to patients with B deficiency. The preferred route of administration depends on the severity of deficiency and patient-specific factors (eg, patients with altered Gastrointestinal absorption of vitamin B may not benefit from oral supplementation). An effective vitamin B replacement 12 12 management of vitamin B deficiency? 12 12 12 12 12 12 regimen includes 1 mg/d of vitamin B given intramuscularly for 1 week, followed by 1 mg/wk for 4 weeks, and finally 1 mg/mo for life. After vitamin B stores have been augmented by parenteral therapy, oral daily vitamin B supplementation (1-2 mg daily) can be effective in some patients. In this case, zinc and other micronutrient supplementation should also be provided.34 When do the In patients with vitamin B deficiency, evidence of clinical improvement occurs within hours of initiation of effective vitamin B therapy. Megaloblastosis reverses in 12 12 clinical manifestations 24 hours; normal hematopoiesis is established within 48 hours; and anemia begins to improve within a week (normalization requires around 8 weeks).34 deficiency begin of vitamin B 12 to reverse after therapy is initiated?
830
831 Key Points
peripheral nervous system, including the somatic (motor, sensory) Peripheral neuropathy describes injury to any part of the and autonomic nervous systems. Polyneuropathy is a type of generalized peripheral neuropathy
that is distinct from radiculopathy, plexopathy, mononeuropathy, Symptoms of polyneuropathy include varying combinations of and mononeuritis multiplex. numbness, paresthesias, pain, autonomic disturbances, and weakness.
Physical findings of polyneuropathy include varying combinations hyporeflexia,.and autonomic disturbances (eg, orthostatic of hypoesthesia, hyperesthesia, allodynia, sensory ataxia, hypotension) In patients with polyneuropathy, physical examination can identify the pattern of nerve fibers involved, narrowing the Electrodiagnostic studies can be helpful in patientstwith peripheral involved (ie, motor, sensory, or both), and give a sense of the time Tests are available to evaluate the autonomic nervous system (eg, differential diagnosis. neuropathy to localize the lesion, determine the pa tern of nerve injury (ie, axonal or demyelinating), determine the fiber type course and severity of the disease. tilt table testing). Additional studies that can be helpful in select patients with
polyneuropathy include genetic studies, CSF examination, skin Polyneuropathy can be metabolic, toxic, inflammatory, or biopsy, and nerve biopsy. hereditary. Inflammatory polyneuropathy can be infectious or nonin ectious. In general, metabolic and toxic causes of polyneuropathyfresult in hereditary causes of polyneuropathy result in demyelinating axonal neuropathy (with exceptions), whereas inflammatory and neuropathy (with exceptions). The combination of history, physical examination, blood tests, and underlying diagnosis in most cases of polyneuropathy. ancillary tests (eg, electrodiagnostic studies) establishes an
832
833
References 1. Callaghan BC, Price RS,tChen KS, Feldman EL. The importance of rare subtypes in diagnosis and treatmen of peripheral neuropathy: a review. JAMA Neurol. 2015;72(12):1510-1518.
- Hughes RA. Peripheral neuropathy. BMJ. 2002;324(7335):466-469.
-
Poncelet AN. An algorithm for the evaluation of peripheral neuropathy. Am Fam Physician. 1998;57(4):755-764.
-
Callaghan BC, Price:RS, Feldman EL. Distal symmetric polyneuropathy: a review. 5. Bowen BC, Seidenwurm DJ. Expert panel on neurologic I. Plexopathy. AJNR Am J JAMA. 2015;314(20) 2172-2181. Neuroradiol. 2008;29(2):400-402.
-
Finsterer J. Systemic and non-systemic vasculitis affecting the peripheral nerves. Acta Neurol Belg. 2009;109(2):100-113.
-
Misra UK, Kalita J, Nair PP. Diagnostic approach to peripheral neuropathy. Ann Indian Acad Neurol. 2008;11(2):89-97.
-
Ropper AH, Samuels MA, Klein JP, eds. Adam and Victor’s Principles of Neurology. 10th ed. China: McGraw-Hill Education; 2014.
-
Feinberg J. EMG: myths and facts. HSS J. 2006;2(1):19-21.
- Watson JC, Dyck PJ. Peripherallneuropathy: a practical approach to diagnosis and symptom management. Mayo C in Proc. 2015;90(7):940-951.
-
Krishnan AV, Kiernan MC. Neurological complications of chronic kidney disease. Nat Rev Neurol. 2009;5(10):542-551.
-
Beghi E, Delodovici ML, Bogliun G, et al.:Hypothyroidism and polyneuropathy. J Neurol Neurosurg Psychiatry. 1989;52(12) 1420-1423.
- Hammond N, Wang Y, Dimachkie MM, Barohn RJ. Nutritional neuropathies. Neurol Clin. 2013;31(2):477-489. . Chaudhry V, Corse AM, O’Brian R, Cornblath DR, Klein AS, Thuluvath PJ.
14Autonomic and peripheral (sensorimotor) neuropathy in chronic liver disease: a clinical and electrophysiologic study. Hepatology. 1999;29(6):1698-1703.
- Stewart BM. The hypertrophic neuropathy of acromegaly; a.rare neuropathy associated with acromegaly. Arch Neurol. 1966;14(1):107-110
-
Hermans G, De Jonghe B, Bruyninckx F, Van den Berghe G. Clinical review: critical illness polyneuropathy and myopathy. Crit Care. 2008;12(6):238.
-
Staff NP, Windebank AJ. Peripheral neuropathy due to vitamin deficiency, toxins, and medications. Continuum (Minneap Minn). 2014;20(5 Peripheral Nervous System
Disorders):1293-1306. 18. Nascimento OJ. Leprosy neuropathy: clinical presentations. Arq Neuropsiquiatr. 2013;71(9B):661-666.
-
Kaku M, Simpson DM. HIV neuropathy. Curr Opin HIV AIDS. 2014;9(6):521-526. 20. Logigian EL, Steere AC. Clinical and electrophysiologic findings in chronic neuropathy of Lyme disease. Neurology. 1992;42(2):303-311.
-
Manikyamba D, Satyavani A, Deepa P. Diphtheritic polyneuropathy in the wake of resurgence of diphtheria. J Pediatr Neurosci. 2015;10(4):331-334.
-
Shin SC, Robinson-Papp J. Amyloid neuropathies. Mt Sinai J Med. 2012;79(6):733-748. 23. Yuki N, Hartung HP. Guillain-Barre syndrome. N Engl J Med. 2012;366(24):2294-2304.
834
- Zivkovic:SA, Lacomis D, Lentzsch S. Paraproteinemic neuropathy. Leuk Lymphoma. 25. Koike H, Sobue G. Paraneoplastic neuropathy. Handb Clin Neurol. 2013;115:713-726. 26. Bougea A, Anagnostou E, Konstantinos G, George P, Triantafyllou N, Kararizou E. A 2009;50(9) 1422-1433. systematic review of peripheral and central nervous system involvement of
rheumatoid arthritis, systemicflupus erythematosus, primary Sjogren’s syndrome, and associated immunological pro iles. Int J Chronic Dis. 2015;2015:910352. . Vallat JM, Sommer C, Magy L. Chronic inflammatory demyelinating 27polyradiculoneuropathy:.diagnostic and therapeutic challenges for a treatable condition. Lancet Neurol 2010;9(4):402-412. 28. Hoyle JC, Jablonski C, Newton HB.iNeurosarcoidosis: clinical review of a disorder with challenging inpatient presentat ons and diagnostic considerations. Neurohospitalist. 2014;4(2):94-101.
- Adam MP, Ardinger HH, Pagon RA, et al, eds. GeneReviews(R). Seattle, WA; 1993.
- Hoyle JC, Isfort MC, Roggenbuck J, Arnold WD. The genetics of Charcot-Marie-Tooth disease: current trends and future implications for diagnosis and management. Appl
Clin Genet. 2015;8:235-243. 31. Fridman V, Oaklander AL, David WS, et al. Natural history and biomarkers in hereditary sensory neuropathy type 1. Muscle Nerve. 2015;51(4):489-495.
-
Tracy JA, Dyck PJ. Porphyria and its neurologic manifestations. Handb Clin Neurol. 2014;120:839-849.
-
Menezes MP, Ouvrier RA. Peripheral neuropathy associated with mitochondrial disease in children. Dev Med Child Neurol. 2012;54(5):407-414.
-
Briani C, Dalla Torre C, Citton V, et al. Cobalamin deficiency: clinical picture and radiological findings. Nutrients. 2013;5(11):4521-4539. . Matchar DB, McCrory DC, Mill ngton DS, Feussner JR. Performance of the serum
35cobalamin assay for diagnosis oficobalamin deficiency. Am J Med Sci. 1994;308(5):276- 283.
835
CHAPTER 42
836
Seizure
837 Case: A 68-year-old man with pronator drift afA previously healthy 68-year-old man is admitted to the hospital flexed his right arm and.leg; moments later the patient turned his head was unresponsive. The patient was confused for 15 minutes afterward, unintentional,weight loss of 18 pounds. The patient underwent a tooth seTemperature is 37.7°C. The patient appears cachectic. There is a 3/6 axilla. Cranial nerves are intact. There is pronator drift of the left demonstrates multiple ring-enhancing cystic lesionsiinvolving thews 2.2 ×2.1 cm. ter his wife witnessed an unusual event. The two were eating breakfast when the patient suddenly extended his left arm and leg and and eyes toward the left This was followed by rhythmic flexion and extension of his arms and legs for 2 minutes, during which time he and does not recall much of the event. He has not been feeling well for a few months and endorses malaise, drenching night sweats, and an extraction procedure 6 months ago. He has no prior history of izures. holosystolic murmur best heard over the apex with radiation to the upper extremity. Reflexes and sensation are intact. Magnetic resonance imaging (MRI) of the brain w th axial (Figure 42-1A and C), coronal (Figure 42-B) and sagittal (Figure 42-1D) vie bilateral frontal and temporal lobes, the largest of which is
FIGURE 42-1
What is the most likely cause of seizure in this patient?
What is a seizure?
What conditions are commonly confused with seizure? What are the symptoms and signs of a seizure?
What is myoclonus?
What is tonic activity?
What is clonic activity?
What is tonic-clonic
A seizure is defined as abnormal excessive or synchronous neuronal activity in the brain resulting in transient symptoms and signs.1 Migraine headache, syncope, transient ischemic attack, psychogenic seizure, Ménière’s disease, and movement disorders can be confused with seizure.2 The clinical manifestations of seizures vary by type and contributing factors but can involve disturbances in sensory, motor, and autonomic function, consciousness, emotional state, memory, cognition, and behavior. These features can occur in isolation or in combination. Common manifestations include myoclonic jerking, alteration of awareness, tonic or clonic activity (or both), atonic activity, automatisms, tongue biting, incontinence, and postictal confusion.2,3 Myoclonus describes sudden-onset, brief (<100 ms), and involuntary isolated or repetitive contraction of a muscle or groups of muscles.4 Tonic activity describes an increase in muscle contraction that is sustained and can last from a few seconds to several minutes.4 Clonic activity describes myoclonus that is regular, repetitive (at a rate of 2-3 contractions per second), and sustained for prolonged periods of time, involving the same muscle groups.4 Tonic-clonic activity describes the paired sequence of tonic movement followed by clonic movement.4
838
activity? What is atonic activity?
What is dystonic activity? What are automatisms?
What is an aura?
What is status epilepticus? What is a postictal state?
What is the role of physical examination in the evaluation of a patient with a suspected seizure disorder? What is the role of electroencephalography (EEG) in the evaluation of a patient with a possible seizure disorder? How sensitive is electroencephalography in epileptic patients?
What supplementary studies can be useful in some patients with suspected seizure? When is emergency neuroimaging recommended after a first seizure?
Atonic activity describes sudden loss of or Decreased muscle tone involving the head, trunk, jaw, or limb musculature without preceding myoclonic or tonic activity, lasting ≥1 to 2 seconds.4 Dystonic activity describes simultaneous sustained contraction of agonist and antagonist muscles, resulting in athetoid or twisting movements, which may produce abnormal postures.4 Automatisms are coordinated, repetitive, motor activity (eg, lip smacking) that can resemble voluntary movement but occur when cognition is impaired. Afterward, the patient is usually amnestic.4 Aura describes an ictal symptom (eg, visual disturbance) that occurs without a change in awareness and may precede an observable seizure. When it occurs in isolation, an aura is considered to be a focal sensory seizure.4 Status epilepticus is reached when the duration of a typical seizure is longer than expected (usually
5 minutes), or when seizures recur in succession without interval return to baseline consciousness.4,5 A postictal state describes the period immediately following a seizure characterized by changes in behavior, motor function, and neuropsychological performance, lasting until baseline status returns, which may take anywhere from seconds to days.6 Physical examination can identify neurologic deficits and findings indicative of systemic disease (eg, lymphadenopathy), which can be suggestive of an underlying cause of the seizure.2
EEG can confirm the presence of abnormal electrical activity, determine the location of the seizure focus, and provide information about the type of seizure disorder.2
The initial EEG is normal in around one-half of patients with epilepsy. If suspicion remains, the EEG should be repeated after the patient has been deprived of sleep (which lowers the seizure threshold). In a small proportion of epileptic patients, electroencephalographic abnormalities cannot be detected despite multiple EEGs.2 Blood tests (eg, basic metabolic panel, serologies), cerebrospinal fluid evaluation, and neuroimaging (eg, brain MRI) can be useful in select patients with a suspected seizure disorder.2
Emergency neuroimaging should be performed after a first seizure in patients on therapeutic anticoagulation, and in those with new focal neurologic deficits, persistent delirium, fever, recent trauma, persistent headache, a history of malignancy, and risk factors for acquired immunodeficiency syndrome.2
What is the pharmacologic treatment of choice for an active seizure? What are the 2 general categories of seizures?
Most seizures resolve spontaneously within a few minutes. Immediately identifiable and reversible etiologies (eg, hypoglycemia) should be treated. If the seizure is prolonged or status epilepticus has been reached, and no reversible cause is apparent, then first-line treatment is a short-acting benzodiazepine administered intravenously.5 Seizures can be unprovoked or provoked.
What is an An unprovoked seizure occurs in the absence of a temporary or reversible associated condition.1 unprovoked seizure? What is a provoked A provoked seizure occurs in close temporal relationship with an acquired precipitating condition; these seizures seizure? are also referred to as “acute symptomatic seizures.”1,7,8 For the purposes of the framework in this chapter, seizures associated with acquired conditions are categorized as provoked. However, conditions that result in residual brain alteration (eg, head trauma, stroke) or those that are irreversible (eg, brain tumor) can also lead to a long-term predisposition for “unprovoked” seizures outside of the acute period (ie, “remote symptomatic seizures”).3,7,8 After a single Following a single unprovoked seizure, the risk for a second seizure over the next 2 to 8 years is approximately 40% to unprovoked 50%.1 seizure, what is the risk of a second seizure? After 2 Following 2 unprovoked seizures, the risk for a third seizure over the next 4 years is approximately 60% to 90%.1 unprovoked seizures, what is the risk of a third seizure?
839
epilepsy? Epilepsy is a disorder of the brain characterized by recurrent unprovoked seizures (at least 2 seizures occurring >24 hours What is apart), or 1 unprovoked seizure with a high probability for a second seizure (≥60% over the next 10 years). The likelihood of a second seizure is based on risk factors such as permanent structural disease of the brain. Patients with epilepsy syndromes (eg, juvenile myoclonic epilepsy) experience seizures along with other characteristic clinical, genetic, and electroencephalographic features.1
The seizure threshold is generally lower in epileptics, and as such, these patients are at higher risk for provoked seizures compared with nonepileptic patients under similar seizure-precipitating conditions.
How common is epilepsy? Is epilepsy a lifelong disorder?
What are the causes of epilepsy?
What are the triggers for seizure activity in epileptic patients? What are the nonpharmacologic long-term treatment options for patients with epilepsy? How effective are antiepileptic drugs in the long-term treatment of epilepsy? What are the 2 main subtypes of seizures in patients with epilepsy?
What general regions of the brain are affected by focal and generalized seizures?
Epilepsy is a common neurologic condition worldwide; approximately 3% of the general population in the industrialized world will have epilepsy at some point during life.2 Epilepsy can be a lifelong disorder. However, it is considered to be resolved when patients with age-related epilepsy syndromes are outside of the age range or when patients are seizure-free for at least 10 years without antiepileptic medications for the most recent 5 years.1 In most cases, epilepsy is either genetic (ie, due to a known mutation) or “idiopathic” (these cases are most likely due to undiscovered genetic conditions). Residual brain alteration from acute conditions, such as head trauma, stroke, brain tumors, and intracranial infection, can lead to a long-term predisposition for seizures and epilepsy.3,9,10 Stimuli such as flashing lights, intense exercise, loud music, and strong emotions can trigger seizures in epileptic patients. Additionally, the seizure threshold in epileptic patients can be lowered by various factors, including lack of sleep, fever, hormonal changes, hyperventilation, stress, and certain medications (eg, bupropion).11 Lifestyle modification (eg, avoiding sleep deprivation, abstaining from alcohol) and behavioral techniques (eg, biofeedback) can effectively reduce the seizure burden for some patients with epilepsy.12
Overall long-term remission is achieved with antiepileptic drugs in most patients with epilepsy. A combination of agents is commonly required, as monotherapy is effective in less than one-half of patients. The choice between the various antiepileptic drugs must take into account patient and drug-related factors.13
Patients with epilepsy can experience focal (ie, “partial”) or generalized seizures.
Focal seizures involve a portion of the brain at onset and are limited to 1 hemisphere; consciousness may or may not be impaired. Generalized seizures involve both cerebral hemispheres simultaneously with resultant loss of consciousness. Focal seizures may become secondarily generalized (ie, focal seizure evolving to a bilateral, convulsive seizure) (Figure 42-2).9,10,14
840
FIGURE 42-2 Pathways of seizure propagation. A, Focal seizure. B, Focal seizure with secondary generalization. C,
Why is it important to distinguish between focal and generalized seizures?
Primary generalized seizure. (Adapted from Golan DE, et al. Principles of Pharmacology: The Pathophysiologic Basis of Drug Therapy. 3rd ed. Philadelphia, PA: Lippincott Williams & Wilkins; 2011.)
Distinguishing between focal and generalized seizures is needed to direct strategies for therapy. Some antiepileptic agents specifically treat focal seizure disorders but are ineffective for generalized seizure disorders (and may even lower the seizure threshold in these patients). Other agents treat generalized seizure disorders but are ineffective for focal seizure disorders. Furthermore, there are surgical treatment options for some types of focal, but not generalized, seizure disorders.15
841
Focal Seizures
What are the 2 main subtypes of focal seizures?
A fully alert 18-year-old man experiences sudden stiffening of the right arm that persists for 1 minute. During a lecture, a 43-year-old algebra teacher suddenly appears to stare into space and then begins to rhythmically pick at thin air for 45 seconds; the students are unable to get her attention, and she is confused for several minutes after the event.
What is the principle difference between simple and complex focal seizures? Why is it important to distinguish between simple simple focal and complex focal seizures? What are the clinical manifestations of focal-onset seizures?
What is Todd’s paralysis?
How effective is pharmacologic monotherapy for patients with a focal seizure disorder? Which antiepileptic drugs are effective as initial monotherapy in adult patients with focal seizure disorders?
Which antiepileptic drugs are effective as initial monotherapy in elderly patients with focal seizure disorders?
Simple focal seizure (ie, focal seizure without impairment of consciousness).
Complex focal seizure (ie, focal seizure with impairment of consciousness).
Consciousness is impaired in complex focal seizures but preserved in simple focal seizures.
The distinction between simple focal and complex focal seizures is needed to accurately assess seizure risk and patient safety. Patients with complex focal seizure disorders may be subject to activity restrictions, including driving and the use of machinery.12 Depending on the region of the brain that is affected, focal-onset seizures can present with a variety of manifestations including motor (eg, focal limb movement), somatosensory (eg, paresthesia), special sensory (eg, visual, auditory, olfactory, gustatory, vertiginous), autonomic (eg, sweating), and psychic (eg, fear). For example, seizures originating from the temporal lobe often present with lip smacking or other oral and alimentary automatisms, whereas those originating from the occipital lobe present with visual symptoms. Manifestations may spread to adjacent areas of the body when the motor cortex is involved (ie, Jacksonian march).12,16 Temporary postictal weakness in the region of the body affected by a focal motor seizure is referred to as Todd’s paralysis.12 Pharmacologic monotherapy is effective in around one-third of patients with a focal seizure disorder. Combination therapy is often necessary.13 Initial monotherapy options with established efficacy for adult patients with focal seizure disorders include carbamazepine, levetiracetam, phenytoin, and zonisamide. Options that are probably or possibly efficacious include valproic acid, gabapentin, lamotrigine, oxcarbazepine, phenobarbital, topiramate, and vigabatrin. Lacosamide is an effective and well-tolerated adjuvant agent in patients with focal seizure disorders.13,17 Initial monotherapy options with established efficacy for elderly patients with focal seizure disorders include gabapentin and lamotrigine.17
842
Generalized Seizures
What are the main subtypes of generalized seizures?
A 16-year-old boy is evaluated for recurrent episodes of staring lasting up to 10 seconds, during which time he is unresponsive. After the episodes he is alert and oriented. The events are reproduced when the patient is asked to hyperventilate. A 33-year-old woman with early morning episodes of sudden-onset, brief, and involuntary dysrhythmic contractions of the proximal limbs. A 54-year-old man is evaluated after his wife complains of recurrent episodes of sudden-onset flexion of the waist for around 15 seconds during sleep. Rhythmic contractions of the shoulder muscles at a rate of 2 to 3 per second followed by confusion. A 46-year-old woman is evaluated for an episode of sudden loss of postural tone resulting in a fall and is found to have an abnormal EEG. A 28-year-old man is observed suddenly stiffening, then jerking the extremities for nearly 2 minutes; he is confused after the event.
Which antiepileptic drugs are effective as initial monotherapy in patients with absence seizures?
Which antiepileptic drugs may precipitate or aggravate myoclonic seizures in susceptible patients?
What is the typical duration of tonic seizures? What is the difference between myoclonic and clonic seizures?
What is the typical duration of atonic seizures?
Which antiepileptic drugs are typically used as initial monotherapy in adults with generalized tonic-clonic seizure disorders?
Absence seizure (ie, petit mal seizure).
Myoclonic seizure.
Tonic seizure.
Clonic seizure. Atonic seizure.
Tonic-clonic seizure (ie, grand mal seizure).
Initial monotherapy options with established efficacy for patients with absence seizures include ethosuximide and valproic acid. Lamotrigine is an option with possible efficacy.17 Myoclonic seizures can be precipitated or aggravated by carbamazepine, gabapentin, oxcarbazepine, phenytoin, tiagabine, and vigabatrin.17 Tonic seizures typically last for 15 to 20 seconds.12 Clonic seizures are characterized by sustained and rhythmic myoclonus and are usually associated with a postictal state.4 Atonic seizures typically last for several seconds and rarely longer than 1 minute.12 Initial monotherapy options with possible efficacy for adult patients with generalized tonic-clonic seizure disorders include carbamazepine, lamotrigine, oxcarbazepine, phenobarbital, phenytoin, topiramate, levetiracetam, and valproic acid.17
843
Provoked Seizures
How common are provoked seizures? The causes of provoked seizures can be separated into which general subcategories?
Are provoked seizures usually generalized or focal?
Provoked seizures account for almost one-half of all first seizures.8 The causes of provoked seizures can be separated into the following subcategories: vascular, toxic, structural, infectious, and metabolic.
Provoked seizures related to nonstructural conditions typically present as generalized seizures (eg, hypoglycemia is usually associated with generalized tonic-clonic seizures). However, provoked seizures related to structural conditions (eg, cerebral aneurysm) typically present as focal seizures.
844
Vascular Causes of Seizure
What are the vascular causes of seizure?
845
846
847
848
Toxic Causes of Seizure
What are the toxic causes of seizure?
Seizures related to this substance occur more commonly as a result of withdrawal than intoxication. Iatrogenic. These substances are ingested, snorted, inhaled, and injected. A 29-year-old man is found with seizurelike activity inside his garage with his car running.
What are the characteristics of seizures related to alcohol withdrawal? Intoxication with which medications are associated with the highest risk for seizure? Seizures can occur as a result of withdrawal from what commonly prescribed medications? Seizures can occur with intoxication from which recreational drugs? What treatment should be given as soon as possible to patients with carbon monoxide poisoning?
Alcohol.
Medication (including intoxication and withdrawal). Recreational drugs.
Carbon monoxide poisoning.
Alcohol withdrawal seizures are common, constituting around one-third of total hospital admissions related to seizures. Onset is typically 6 to 48 hours after cessation of alcohol and the seizure type is usually generalized.8 The tricyclic antidepressants, such as maprotiline, clomipramine, and amoxapine, are associated with a high rate of seizures when therapeutic levels are exceeded (seizure occurs up to one-fifth of patients who overdose). Other medications associated with seizures include theophylline, isoniazid, cyclosporine, chlorpromazine, clozapine, bupropion, meperidine, and flumazenil.8 Withdrawal from barbiturates, benzodiazepines, and other sedatives (eg, zolpidem) can result in seizures.8
Intoxication with cocaine, amphetamines, and hallucinogens can result in seizures.8
In patients with carbon monoxide poisoning, oxygen should be given at a concentration of 100% via either a nonrebreather mask or hyperbaric chamber.
849
Structural Causes of Seizure
What are the structural causes of seizure?
A 64-year-old man with a history of non– small cell lung cancer, thought to be in remission, presents with a focal complex seizure characterized by lip smacking. A 76-year-old woman experiences a focal complex seizure 8 hours after being admitted to the hospital following a ground-level fall and is found to have normal neuroimaging.
How often is seizure the presenting clinical sign of a brain tumor? Which inherited condition that results in the development of brain tumors should be suspected in a patient with seizures and multiple hypopigmented macules? What is the risk of developing epilepsy after traumatic brain injury?
Metastatic brain tumor.
Traumatic brain injury (TBI).
Seizures are the presenting manifestation of brain tumors in up to one-half of patients and are more common with low-grade tumors.22 Tuberous sclerosis complex should be suspected in a patient with a seizure disorder who is found to have hypopigmented macules (ie, ash leaf spots) on physical examination.
The risk of developing epilepsy (ie, future seizures outside of the acute setting) after TBI increases with severity of injury and is higher in patients who experience acute seizure. Epilepsy is approximately 15 times more common in adult patients with severe TBI compared with the general population. Most develop epilepsy within the first year after the event, but it may occur many years later.8
850
Infectious Causes of Seizure
What are the infectious causes of seizure?
A 20-year-old college student presents with headache, photophobia, neck stiffness, and a generalized seizure. A 55-year-old woman with a history of kidney transplant presents with confusion and headache and is found to have periodic lateralized epileptiform discharges (ie, lateralized periodic discharges) on EEG. A 48-year-old man with an untreated dental abscess presents with a complex focal seizure. Parasites. A viral-mediated demyelinating condition of the central nervous system found in immunocompromised patients.
How often do seizures occur in patients with bacterial meningitis?
How often do seizures occur in patients with herpes simplex encephalitis?
How often do seizures occur in patients with brain abscess?
What is the risk of seizure related to toxoplasmosis?
What is the risk of epilepsy in patients with neurocysticercosis who have persistent abnormalities on neuroimaging (eg, cysts and calcified cysts)?
What are the characteristics of seizures related to progressive multifocal leukoencephalopathy?
Meningitis. Herpes simplex or human herpesvirus 6 encephalitis.
Brain abscess. Toxoplasmosis and neurocysticercosis. Progressive multifocal leukoencephalopathy (PML).
Seizures occur in around one-fifth of patients with community-acquired bacterial meningitis; when seizures do occur, there is a significantly Increased risk of mortality.23 Seizures are part of the presenting manifestations in about one-half of patients with herpes simplex encephalitis, probably as a result of involvement of the frontotemporal cortex, which is highly epileptogenic.24 Seizures are the presenting manifestation in around one-quarter of patients with brain abscess. Epilepsy subsequently develops in a significant proportion of patients.25,26 It is estimated that one-third of the world’s population is infected with toxoplasmosis. There is an associated 1.5- to 4-fold risk of seizure in these patients.27 Epilepsy develops in just over one-half of patients with neurocysticercosis who have persistent abnormalities on neuroimaging. Neurocysticercosis is the most common cause of epilepsy in the developing world, responsible for around one-third of all seizures.8 Seizures occur in around one-fifth of patients with PML. The most frequent seizure types include simple focal, complex focal, and simple focal with secondary generalization. Seizures are generally responsive to antiepileptic drugs and do not affect survival.28
851
Metabolic Causes of Seizure
What are the metabolic causes of seizure?
This condition should be ruled out in the immediate evaluation of anyone with impaired consciousness. Hypertonicity and polyuria in a diabetic patient. Seizure in a marathon runner who rehydrates with water. Seizure in a patient with diabetes insipidus. These electrolyte disturbances are associated with Chvostek’s sign. Tissue death. Delirium, jaundice, and coagulopathy. A 28-year-old woman who is 32 weeks pregnant presents with hypertension, proteinuria, and a generalized tonic-clonic seizure. Tremor, hyperreflexia, and a generalized seizure. Acute episodes of abdominal pain, polyneuropathy, and urine that turns red over time (Figure 42-4).
Hypoglycemia.
Hyperglycemia. Hyponatremia. Hypernatremia. Hypocalcemia and hypomagnesemia. Cerebral hypoxia with anoxic brain injury. Liver failure. Eclampsia.
Hyperthyroidism. Acute porphyria.
FIGURE 42-4 Urine from a patient with porphyria cutanea tarda (right) and from a patient with normal porphyrin excretion (left). From Champe PC, Harvey RA, Ferrier DR. Biochemistry. 4th ed. Philadelphia, PA: Wolters Kluwer Health/Lippincott Williams & Wilkins; 2008, with permission.
What is the most common type of seizure related to hypoglycemia? Are seizures more common in patients with diabetic ketoacidosis or nonketotic hyperosmolar state? What treatment should be immediately administered to patients with seizures related to hyponatremia? Under what circumstance do seizures occur most often in patients with hypernatremia? What degree of hypomagnesemia is generally required to cause
Patients with hypoglycemia most often experience generalized tonic- clonic seizures.29 Seizures tend to occur in patients with nonketotic hyperosmolar state but are rare in patients with diabetic ketoacidosis, likely related to the anticonvulsant effect of ketosis. Focal motor seizure is the most common type.29 Intravenous hypertonic saline (3%) should be given immediately to patients with seizures related to hyponatremia.30
Seizures related to hypernatremia most commonly occur when chronic hypernatremia is corrected too quickly.30 Seizures generally occur in association with severe hypomagnesemia (<1 mEq/L) and are usually generalized tonic-clonic.30
852
seizures? How often do seizures occur in Seizures occur in around one-half of patients with symptomatic hypoparathyroidism.30 patients with symptomatic hypoparathyroidism? What conditions can lead to Cerebral hypoxia can occur with any of the following conditions: impaired arterial oxygen content cerebral hypoxia? (eg, anemia, hypoxemia), impaired oxygen delivery (eg, low cardiac output), impaired binding of oxygen to hemoglobin (eg, methemoglobinemia), and impaired uptake or use of oxygen by the tissues (eg, carbon monoxide poisoning, sepsis, cyanide poisoning). What is dialysis disequilibrium DDS may occur when rapid removal of urea during hemodialysis results in an osmotic gradient, syndrome (DDS)? leading to cerebral edema. Manifestations include nausea, headache, vomiting, tremor, and seizure.31 What type of liver disease is Seizures can occur in patients with acute (ie, fulminant) liver failure, probably related to the cerebral associated with seizures? edema that can develop with this condition, but are an unusual manifestation of chronic liver disease.32 When do seizures usually occur Seizures associated with eclampsia occur prepartum in around one-half of patients and postpartum in in patients with eclampsia? the other half. A small proportion of patients experience seizures during childbirth.8 Which antiepileptic drugs can Antiepileptic drugs that can worsen an acute attack of porphyria include phenytoin, phenobarbital, potentially worsen an acute attack clonazepam, and valproic acid.33 of porphyria?
853 Case Summary A 68-year-old man presents with a first-time seizure after several
months of malaise, night sweats, and weight loss andfis found to have a low-grade fever, heart murmur, focal neurologic de icit, and multiple ring-enhancing lesions on neuroimaging.
What is the most likely cause of seizure in this patient? Brain abscess.
854
What type of seizure i What region of the brain epileptic focus in this The “fencing” posturing described in this case (extending the arm and leg on one side while flexing the contralateral limbs) is reflect ve of a focus in the foci). The most likely culprit lesion in this case is in the right frontal lobe (see Figure 41-1). Localizing seizure foci based only on clinical history and explored. neurologic finding (ie, pronator drift). Bonus Questions
described in this case?s is likely to have been the case?
case suggest a provoked What are the What features of this seizure? mechanisms through which brain abscesses occur? What are some examples
of regional infection that can lead to brain abscess via contiguous spread? What is the most likely abscess in this case? culture can be collected mechanism of brain What specimens for in patients with brain abscess to identify)the How are brain abscesses culprit organism(s ? managed? What dangerous associated with brain What is the prognosis of complications are abscess? patients with brain abscess?
The patient in this case experienced a focal seizure with secondary generalization (ie, focal seizure evolving to a bilateral, convulsive seizure). supplementary motor area of the frontal lobe; contralateral gaze deviation is also classically seen with frontal lobe foci (it can also beiseen with temporal lobe examination can be challenging, and EEG is usually necessary. Localizing the origin of focal seizures is important when surgical treatment options are being In this case, features suggestive of a provoked seizure include older age of onset, systemic symptoms and signs (eg, night sweats, weight loss), and the focal
Brain abscesses form via contiguous spread through disrupted barrierstsurrounding the brain in around one-half of cases; hematogenous spread is the mechanism in one-quarter of cases; and the mechanism is unknown in he remaining quarter of cases. 25
Brain abscess is a potential complication of otitis media, mastoiditis, sinusitis, and dental abscess.25
Multiple bilateral foci on neuroimaging inithis case are suggestive of hematogenous spread. The presence of a holosystolic murmur and preceding dental The causative organism of a brain abscess can be identified via blood and cerebrospinal fluid cultures in around one-quarter of cases. Caution should be Ultimately, neurosurgical stereotactic aspiration may be necessary to obtain positive cultures. Almost any brain abscess >1 cm in diameter is amenable to this work suggest subacute bacterial endocard tis. exercised before performing lumbar puncture in these patients given the risk of brain herniation. Regional infections, if present, should be cultured. technique. Gram stain and aerobic and anaerobic cultures should be obtained. Other specific studies (eg, acid-fast smear) should be obtained depending on patient-specific factors.25 A combination of neurosurgical drainage and intravenous antibiotics is typically used to treat brain abscesses. The course of intravenous antibiotics is often long (about 6-8 weeks). Serial imaging studies should be performed to monitor clinical response and guide the duration of therapy. 25 In patients with brain abscess, ventriculitisican occur from abscess rupture into the ventricular system and is associated with a high mortality rate. Other dangerous complications include status ep lepticus, hydrocephalus, and brain herniation. 25
Overall, good outcomes with no or minimal long-term sequelae are observed in most patients treated for brain abscess. However, epilepsy develops in a significant proportion of patients. 25,26
855 Key Points
neuronal activity resulting in transient symptoms and signs, Status epilepticus is reached when the duration of a typical seizure recur in succession without interval return to baseline A seizure is defined as abnormal excessive or synchronous including sensory, motor, and autonomic manifestations. is longer than expected (usually >5 minutes), or when seizures consciousness. A postictal state is the transient period after a seizure characterized performance. by changes in behavior, motor function, and neuropsychological Seizures can be focal or generalized. Focal seizures involve a portion of the brain at onset and are cerebralihemispheres simultaneously with loss of consciousness. Focal seizures can become secondarily generalized (ie, focal limited to 1 hemisphere. Generalized seizures involve both Focal se zures can be simple (without impaired awareness) or complex (with impaired awareness). seizure evolving to a bilateral, convulsive seizure). Seizures can be unprovoked or provoked.
An unprovoked seizure is one that occurs in the absence of a Epilepsy is a disorder of the brain characterized by recurrent Long-term remission is achieved with antiepileptic drugs in most temporary or reversible precipitant. unprovoked seizures. patients with epilepsy. The causes of provoked seizures can be separated into the
following subcategories: vascular, toxic, structural, infectious, and Causes of provoked seizures that result in residual brain alteration lead to a long-term predisposition for seizures and epilepsy. underlying cause if possible. Antiepileptic medications may be metabolic. (eg, stroke) or those that are not temporary (eg, brain tumor) can Treatment for provoked seizures should focus on reversing the necessary at least temporarily. Most acute seizures resolve spontaneously within a few minutes a seizure is prolonged or status epilepticus is reached, then first-. If short-acting benzodiazepine (unless there is an immediately line pharmacologic therapy is an intravenously-administered reversible provoking condition such as hypoglycemia).
856
857
References 1. Fisher RS, Acevedo C, Arzimanoglou A, et al. ILAE official report: a practical clinical definition of epilepsy. Epilepsia. 2014;55(4):475-482.
- Browne TR, Holmes GL. Epilepsy. N Engl J Med. 2001;344(15):1145-1151.
- definitions proposed by theiInternational LeaguelAgainst Epilepsy (ILAE) and the Fisher RS, van Emde Boas W, Blume W, et al. Epi eptic seizures and epilepsy: International Bureau for Ep lepsy (IBE). Epilepsia. 2005;46(4):470-472.
- Blume WT, Luders HO, Mizrahi E, Tassinari C, van Emde Boas W, Engel J Jr. Glossary of descriptive terminology for ictal semiology: report of the ILAE task force on
classification and terminology. Epilepsia. 2001;42(9):1212-1218. 5. Brophy GM, Bell R, Claassen J, et al. Guidelines for the evaluation and management of status epilepticus. Neurocrit Care. 2012;17(1):3-23.
-
Remi J, Noachtar S. Clinical features of the postictal state: correlation with seizure variables. Epilepsy Behav. 2010;19(2):114-117.
-
Beghi E, Carpio A, Forsgren L, et al. Recommendation for a definition of acute symptomatic seizure. Epilepsia. 2010;51(4):671-675.
-
Beleza P. Acute symptomatic seizures: a clinically oriented review. Neurologist. 2012;18(3):109-119.
-
Berg AT, Berkovic SF, Brodie MJ, et al. Revised terminology and concepts for organization of seizures and epilepsies: report of the ILAE Commission on
Classification and Terminology, 2005-2009. Epilepsia. 2010;51(4):676-685. 10. Sander JW, Hart YM, Johnson AL, Shorvon SD. National General Practice Study of Epilepsy: newly diagnosed epileptic seizures in a general population. Lancet.
1990;336(8726):1267-1271. 11precipitating,factors do patients with epilepsy mostlfrequently report? Epilepsy Behav. . Nakken KO Solaas MH, Kjeldsen MJ, Friis ML, Pe lock JM, Corey LA. Which seizure- 2005;6(1):85-89.
-
Bromfield EB, Cavazos JE, Sirven JI, eds. An Introduction to Epilepsy [Internet]. West Hartford, CT: American Epilepsy Society; 2006.
-
Becerra JL, Ojeda J, Corredera E, Ruiz Gimenez J. Review of therapeutic options for adjuvant treatment of focal seizures in epilepsy: focus on lacosamide. CNS Drugs.
2011;25(suppl 1):3-16. 14. Chang BS, Lowenstein DH. Epilepsy. N Engl J Med. 2003;349(13):1257-1266. 15. Luders HO, Turnbull J, Kaffashi F. Are the dichotomies generalized versus focal epilepsies and idiopathic versus symptomatic epilepsies still valid in modern
epileptology? Epilepsia. 2009;50(6):1336-1343. 16. Ahmed SN, Spencer SS. An approach to the evaluation of a patient for seizures and epilepsy. WMJ. 2004;103(1):49-55. . Glauser T, Ben-Menachem E, Bourgeois B, et al. Updated ILAE evidence review
17antiepileptic drug efficacy and effectiveness as initial monotherapy for epileptic of seizures and syndromes. Epilepsia. 2013;54(3):551-563.
-
Cosgrove GR. Occult vascular malformations and seizures. Neurosurg Clin N Am. 1999;10(3):527-535.
-
Kamali AW, Cockerell OC, Butlar P. Aneurysms and epilepsy: an increasingly recognised cause. Seizure. 2004;13(1):40-44.
-
Hajj-Ali RA, Calabrese LH. Primary angiitis of the central nervous system.
858
Autoimmun Rev. 2013;12(4):463-466. 21. Scott RM, Smith ER. Moyamoya disease and moyamoya syndrome. N Engl J Med. 2009;360(12):1226-1237. . van Breemen MS, Wilms EB, Vecht CJ. Epilepsy in patients with brain tumours:
22epidemiology, mechanisms, and management. Lancet Neurol. 2007;6(5):421-430. 23. Zoons E, Weisfelt M, de Gans J, et al. Seizures in adults with bacterial meningitis. Neurology. 2008;70(22 Pt 2):2109-2115.
-
Misra UK, Tan CT, Kalita J. Viral encephalitis and epilepsy. Epilepsia. 2008;49(suppl 6):13-18.
-
Brouwer MC, Tunkel AR, McKhann GM 2nd, van de Beek D. Brain abscess. N Engl J Med. 2014;371(5):447-456.
-
Muzumdar D, Jhawar S, Goel A. Brain abscess: an overview. Int J Surg. 2011;9(2):136- 144.
-
Ngoungou EB, Bhalla D, Nzoghe A, Darde ML, Preux PM. Toxoplasmosis and epilepsy–systematic review and meta analysis. PLoS Negl Trop Dis.
2015;9(2):e0003525. 28. Lima MA, Drislane FW, Koralnik IJ. Seizures and their:outcome in progressive 29. Verrotti A, Scaparrotta A, Olivieri C, Chiarelli F. Seizures and type 1 diabetes multifocal leukoencephalopathy. Neurology. 2006;66(2) 262-264. mellitus: current state of knowledge. Eur J Endocrinol. 2012;167(6):749-758. . Castilla-Guerra L, del Carmen Fernandez-Moreno M, Lopez-Chozas JM, Fernande
30Bolanos R. Electrolytes disturbances and seizures. Epilepsia. 2006;47(12):1990-1998.z-31. Patel N, Dalal P, Panesar M. Dialysis disequilibrium syndrome: a narrative review. Semin Dial. 2008;21(5):493-498.
- Lewis M, Howdle PD. The neurology of liver failure. QJM. 2003;96(9):623-633.
- Tracy JA, Dyck PJ. Porphyria and its neurologic manifestations. Handb Clin Neurol. 2014;120:839-849.
859
CHAPTER 43
860
Stroke
861 Case: A 48-year-old woman with flank pain with acute-onset severe right-sided flank pain with radiation to theital United States after vacationing in England. She has had left leg combination oral contraceptive pills but no other medications. She is costovertebral angle tenderness is present. Urinalysis demonstratesd e A previously healthy 48-year-old woman is admitted to the hosp back and associated nausea. The patient recently flew home to the swelling and pain since returning home. The patient takes not a smoker. There is erythema and pitting edema of the left lower extremity. Th abdomen is soft, and there is no tenderness to palpation. Right-side hematuria without dysmorphic red blood cells. Doppler ultrasound of the left lower extremity demonstrates
thrombosis of the deep femoral vein. Computed tomography (CT) shows ailack of enhancement of the right kidney (arrow). Shortly after the left and there is a right-sided facial droop. There is full strength of The patient responds to painful stimuli on the left but not the right. differentiation in the territoryiof the left middle cerebral artery. CT imaging with intravenous contrast of the abdomen (Figure 43-1) complet ng the imaging study, the patient suddenly becomes unintelligible. On examination, the head and eyes are deviated toward the left upper and lower extremities but no movement on the right. The biceps, patellar, and Achilles reflexes are brisk on the right. Urgent CT imaging of the bra n shows loss of gray-white matter imaging of the brain 24 hours later is shown in Figure 43-2.
FIGURE 43-1
862
FIGURE 43-2
What is the most likely cause of stroke in this patient?
863
864
Hemorrhagic Stroke
Hemorrhagic strokes occur within which 2 anatomic spaces of the brain?
Hemorrhagic stroke can occur within the brain parenchyma (ie, intracerebral) or subarachnoid space (ie, between the arachnoid and pia mater) (Figure 43-4).
FIGURE 43-4 Hemorrhagic stroke can occur within the brain (intracerebral
Why is it important to differentiate between strokes caused by intracerebral hemorrhage and subarachnoid hemorrhage?
What are the relative rates of intracerebral hemorrhage and subarachnoid hemorrhage?
hemorrhage) or on the surface (subarachnoid hemorrhage). (From Werner R. A Massage Therapist’s Guide to Pathology, 6th ed. Philadelphia, PA: Wolters Kluwer; 2016.)
The underlying causes and general treatment considerations vary between strokes caused by intracerebral hemorrhage and SAH.
Intracerebral hemorrhage is more common than SAH, accounting for 10% to 15% of all strokes. SAH accounts for around 5% of all strokes.5
865
Hemorrhagic Stroke Caused BY Intracerebral Hemorrhage
What risk factors are associated with intracerebral hemorrhage? What are the findings of intracerebral hemorrhage on CT imaging?
Risk factors for intracerebral hemorrhage include hypertension, older age, race (eg, blacks, Asians), high levels of alcohol consumption, and lower levels of low density lipoprotein and triglycerides.6,7
Intracerebral hemorrhage typically appears as a round or oval hyperattenuating lesion on noncontrast CT imaging (Figure 43-5). Early on, the mass measures 40 to 60 Hounsfield units (HU) and can be heterogeneous in appearance. Over time as the clot organizes, it becomes more homogenous and hyperdense, measuring 60 to 80 HU within hours to days and 80 to 100 HU over the course of a few days.5
FIGURE 43-5 Noncontrast CT imaging showing hyperdensity (arrow) in the area of the right temporal lobe corresponding to
What general medical treatment should be considered for acute spontaneous intracerebral hemorrhage? What is the short-term mortality of intracerebral hemorrhage?
the presence of intracerebral hemorrhage. (From Garcia MJ. Noninvasive Cardiovascular Imaging: A Multimodality Approach. Philadelphia, PA: Lippincott Williams & Wilkins; 2010.)
In patients with acute spontaneous intracerebral hemorrhage, underlying bleeding diatheses should be identified and addressed (eg, reversal of elevated international normalized ratio); systolic blood pressure (SBP) should be lowered if elevated (for patients with SBP 150 to 220 mm Hg, it is safe to acutely decrease it to 140 mm Hg); blood sugar should be managed to avoid hyperglycemia or hypoglycemia; and seizures should be treated with antiepileptic medications. In patients with elevated intracranial pressure, additional measures including intubation and sedation, elevation of the head of the bed, hypertonic fluid administration, and hemicraniectomy may be necessary.8
The 30-day mortality of intracerebral hemorrhage is nearly 50%.5
What are the causes of intracerebral hemorrhage?
An S4 gallop on auscultation of the heart and evidence of left ventricular hypertrophy on electrocardiography. A disease of older patients resulting from protein deposition within cerebral blood vessels. A 65-year-old woman suffers a ground-level fall, hitting her head on the pavement, and subsequently develops confusion and focal neurologic deficits. A 29-year-old man is admitted with gingival bleeding, epistaxis, and confusion after a recent upper respiratory tract infection and is found to have a petechial rash on the lower extremities. Congenital lesions. A 38-year-old woman with a recent diagnosis of melanoma presents with focal neurologic deficits. Intracerebral hemorrhage related to this acquired vascular lesion almost always occurs in association with SAH. A nonarterial vascular source of intracerebral hemorrhage. This vascular condition predominates in
Hypertension (HTN).
Cerebral amyloid angiopathy (CAA).
Trauma.
Immune thrombocytopenic purpura.
Cerebral vascular malformation (CVM). Metastatic brain tumor.
Cerebral aneurysm.
Cerebral venous thrombosis (CVT). Moyamoya disease.
866
Asians. A patient’s neurologic status suddenly deteriorates 10 days after an ischemic stroke.
What are the most common locations of intracerebral hemorrhage related to hypertension? Which cerebral vessels are involved in cerebral amyloid angiopathy?
What is the frequency of intracerebral hemorrhage in patients with traumatic brain injury?
Why is it important to determine whether a bleeding diathesis is present in patients with intracerebral hemorrhage? Which types of cerebral vascular malformation are associated with Increased risk of intracerebral hemorrhage? What imaging findings are suggestive of intracerebral hemorrhage associated with underlying tumor?
How often do ruptured cerebral aneurysms result in intracerebral hemorrhage? How common is intracerebral hemorrhage in the setting of cerebral venous thrombosis? What unique imaging finding is characteristic of moyamoya disease?
When does spontaneous hemorrhagic transformation occur during the course of an ischemic stroke?
Hemorrhagic transformation of an ischemic stroke.
Intracerebral hemorrhage related to HTN tends to occur within deep brain structures. The putamen and internal capsule account for most cases; other common sites include the thalamus and pons.5 Amyloid deposition occurs in the small- and medium-sized vessels of the leptomeninges and cortex, while there is relative sparing of the vessels of the basal ganglia, white matter, and posterior fossa. Intracerebral hemorrhage related to CAA tends to be lobar.5 Approximately one-half of patients with traumatic brain injury will develop some type of intracranial hemorrhage. Among those patients, intracerebral hemorrhage occurs in one-half. The risk of mortality increases by a factor of 3 when the size of the intracerebral hemorrhage increases from small to large.9 Identifying a bleeding diathesis in patients with intracerebral hemorrhage represents an opportunity for targeted treatment; diatheses should be corrected when possible.
Arteriovenous malformation (AVM) and cavernous malformation (CM) are associated with intracerebral hemorrhage.5
Intracerebral hemorrhage associated with underlying tumors tend to be more heterogeneous in appearance, slower to degrade over time, associated with thick or nodular enhancement, and associated with a greater degree of surrounding vasogenic edema. Because tumors may be obscured by the hemorrhage, delayed repeat imaging should be considered in patients without an identified alternative cause.5 Up to one-third of ruptured aneurysms result in intracerebral hemorrhage; SAH is also present in the vast majority of these cases.5 Almost half of patients with CVT have an associated intracerebral hemorrhage. The presence of a hyperattenuating cortical or deep vein adjacent to the hematoma on CT imaging is highly suggestive of CVT. Most cases of CVT occur in women.5 Moyamoya disease is characterized by progressive bilateral stenosis of the intracranial portion of the internal carotid arteries and proximal branches, resulting in extensive collateral circulation. This network of collateral arteries resembles a “puff of smoke” on angiography (see Figure 42-3).10 Spontaneous hemorrhagic transformation typically occurs in the second week after ischemic stroke. Early transformation is usually related to thrombolytic therapy or mechanical clot removal.5
867
Hemorrhagic Stroke Caused BY Subarachnoid Hemorrhage
What are the risk factors for subarachnoid hemorrhage? What “warning” symptom may occur in patients with subarachnoid hemorrhage? What are the findings of subarachnoid hemorrhage on CT imaging?
Positive family history of SAH, hypertension, hyperlipidemia, and active smoking are the 4 strongest risk factors for SAH.11 Before the major bleeding event, some patients with SAH experience a sentinel headache, which is described as sudden in onset and unusually severe. The sentinel headache typically occurs within 2 weeks before SAH, with a peak incidence within 1 day.12 SAH typically appears as high attenuation material conforming to the subarachnoid space on noncontrast CT imaging (Figure 43-6). It can be focal or diffuse and present within the sulci, fissures, or basal cisterns. Noncontrast CT imaging is approximately 98% sensitive for SAH within 12 hours, but this decreases to around 93% 24 hours after the event due to the Decreased density of blood over time. A sentinel bleed (a relatively small bleed that precedes the major event) can be missed on imaging.5
FIGURE 43-6 Noncontrast CT imaging showing diffuse subarachnoid hemorrhage (arrows). When it is
Which procedure should be performed when suspicion of subarachnoid hemorrhage remains strong despite negative CT imaging of the head? How can true cerebrospinal fluid erythrocytosis be distinguished from trauma related to needle insertion? What general medical treatment should be considered for acute spontaneous subarachnoid hemorrhage? What is the short-term mortality of subarachnoid hemorrhage?
visualized within the cortical sulci, fissures, and cisterns, hemorrhage is confirmed to be located within the subarachnoid space. (From Pope TL Jr, Harris JH Jr. Harris & Harris’ The Radiologyiof Emergency Medicine. 5th ed. Philadelphia, PA: Lippincott Williams & W lkins; 2013.)
Lumbar puncture with CSF analysis should be performed in patients suspected of SAH when neuroimaging is negative. Typical findings of SAH include elevated opening pressure and CSF erythrocytosis or xanthochromia, depending on how long the blood has been in the subarachnoid space.
When CSF erythrocytosis occurs as a result of trauma, the bloody appearance of the fluid (as well as the CSF red blood cell count) should lessen over time as fluid is collected. The first and last vials provide the most obvious comparison. In patients with acute spontaneous SAH, underlying bleeding diatheses should be identified and addressed, elevated SBP should be lowered carefully (eg, without overly-rapid reduction to a target <160 mm Hg), and oral nimodipine should be administered. In patients with hydrocephalus, extraventricular drain placement should be considered. Additional measures may be necessary in patients with elevated intracranial pressure.13 The 30-day mortality of SAH is approximately 40%.5
What are the causes of subarachnoid hemorrhage?
Usually identifiable from the Craniospinal trauma.5 history, this is the most common overall cause of SAH. The leading cause of Ruptured cerebral aneurysm.5 nontraumatic SAH, accounting for the vast majority of cases. A predictable pattern of bleeding Perimesencephalic nonaneurysmal subarachnoid hemorrhage (PM-NASAH).5 on neuroimaging, centered immediately anterior to the midbrain, with a negative angiographic study.
868
What is the management of minor traumatic subarachnoid hemorrhage? What are the general characteristics of cerebral aneurysms?
What studies are available to evaluate for the presence of a cerebral aneurysm?
What is the prognosis of perimesencephalic nonaneurysmal subarachnoid hemorrhage? What are the rare causes of subarachnoid hemorrhage?
Most cases of traumatic SAH are mild and do not require specific treatment. Serial imaging should be obtained to document stability.5,14 Asymptomatic cerebral aneurysms exist in around 2% of adults. Most are located in the anterior circulation and tend to occur at the branch points of cerebral vessels. Risk of rupture is about 0.7% annually but increases with size, particularly when aneurysms are >7 mm in diameter in the anterior circulation or >5 mm in the posterior circulation.5 Conventional angiography is considered the gold standard for detection of cerebral aneurysms. CT angiography (CTA) and magnetic resonance angiography (MRA) are noninvasive options but are less sensitive, particularly when the aneurysms are small (<3 mm). In the case of a negative initial study, repeat studies are often required, and conventional angiography is the preferred modality.13 Most cases of PM-NASAH are not associated with a cerebral aneurysm and the underlying cause remains elusive. The presentation of PM-NASAH is similar to other types of SAH. However, it is associated with excellent prognosis.5 Rare causes of SAH include vascular malformations (both cerebral and spinal), intracranial arterial dissection, brain tumors, sickle cell anemia, vasculitis, coagulopathy, and pituitary apoplexy.5
869
Ischemic Stroke
Which 2 main arteries supply the brain?
The internal carotid arteries (ICAs) give rise to the anterior circulation, and the vertebral arteries give rise to the posterior circulation. The circle of Willis is an anastomosis between the 2 major systems, allowing for redundant flow when parts of either system become compromised (Figure 43-7).15
FIGURE 43-7 A, The circle of Willis seen from below the brain. B, The arteries of the circle of Willis. (Adapted from Morton PG,
Which major vessels constitute the anterior and posterior circulations? What areas of the brain are supplied by the anterior and posterior systems? What are the 3 subcategories of ischemic stroke?
What are the mechanisms
Fontaine DK. Critical Care Nursing: A Holistic Approach, 10th ed. Philadelphia, PA: Wolters Kluwer Health; 2013.)
The vessels of the anterior circulation include the ICA, middle cerebral artery (MCA), and anterior cerebral artery (ACA). The vessels of the posterior circulation include the vertebral artery, basilar artery, and posterior cerebral artery (PCA).15
The anterior circulation supplies the majority of the cerebral hemispheres, with the exceptions of the occipital and medial temporal lobes, which are fed from the posterior circulation. The posterior circulation supplies the brainstem, cerebellum, and posterior portions of the cerebral hemispheres (ie, occipital and medial temporal lobes).15,16
Ischemic stroke can occur as a result of in situ occlusion, embolism, or watershed infarction.
In all types of ischemic stroke, brain infarction occurs as a result of Decreased cerebral perfusion, with associated Decreased delivery of oxygen and nutrients. In situ occlusive stroke can involve large or small arteries. Large artery
870
of in situ occlusive, embolic, and watershed strokes?
Why is it important to differentiate between the mechanisms of ischemic stroke? In addition to addressing the underlying cause, what general medical treatment should be considered for ischemic stroke related to acute occlusion?
involvement occurs as a result of thrombosis, whereas small artery involvement occurs as a result of other mechanisms (eg, lipohyalinosis). Embolic stroke occurs when debris travels from a remote source and lodges within a cerebral artery. If the source is a cerebral vessel, then the stroke is both thrombotic and embolic. Watershed stroke occurs when a systemic process results in cerebral hypoperfusion that affects the most vulnerable regions of the brain, which are typically the zones between neighboring vascular territories.
Underlying causes and general treatment considerations differ between in situ occlusive, embolic, and watershed strokes.
In patients with ischemic stroke related to acute occlusion, blood pressure should be lowered when it is severely elevated (SBP >220 mm Hg or diastolic blood pressure >120 mm Hg) with a goal of around 15% reduction during the first 24 hours (SBP should be <185/110 mm Hg in patients who are to receive thrombolytics). It is reasonable to try to achieve blood glucose levels in a range of 140 to 180 mg/dL. Intravenous recombinant tissue plasminogen activator (tPA) should be administered to select patients within a limited window of time from onset of ischemic stroke (or from when patient was last seen to be normal if time of onset is unknown). A window of 3 to 4.5 hours is used, depending on patient- specific factors. Mechanical thrombectomy may be beneficial in select patients with large vessel occlusions (eg, terminal ICA, proximal ACA, proximal MCA). Oral administration of aspirin within 24 to 48 hours after stroke (but not within 24 hours of fibrinolytic therapy if it is given) is recommended. Additional measures may be necessary in patients with elevated intracranial pressure, and hemicraniectomy may be beneficial in patients with malignant cerebral edema.17,18
871
Ischemic Stroke Caused by Acute in Situ Occlusion of a Large Vessel
Which cerebral arteries are classified as large vessels? What is the tempo of stroke caused by large vessel thrombosis? What are the most common mechanisms of infarction involving the internal carotid arteries? What clinical syndromes are associated with acute internal carotid artery occlusion?
What clinical syndrome is associated with acute occlusion of the stem of the middle cerebral artery? What clinical syndrome is associated with acute occlusion of the superior division of the middle cerebral artery? What clinical syndrome is associated with acute occlusion of the inferior division of the middle cerebral artery? What clinical syndrome is associated with acute occlusion of the anterior cerebral artery distal to the anterior communicating artery? What clinical syndromes are associated with acute occlusion of the large arteries of the posterior circulation?
Large cerebral arteries include the ICA, vertebral artery, basilar artery, and the circle of Willis and its proximal branches (eg, MCA).
Compared with strokes caused by embolism and small vessel in situ occlusion, large vessel thrombotic strokes are typically associated with a slowly evolving or “stuttering” onset.15
All 3 of the major mechanisms of ischemic stroke, including acute in situ occlusion (from acute thrombosis), embolism, and watershed, can occur in relation to ICA disease. The most common mechanism is acute thrombosis.16
Unlike the more distal large arteries, the clinical syndromes of ICA occlusion are variable and dependent on the constitution of the circle of Willis. When there is a competent circle of Willis (which provides redundant blood flow), no area of the brain is entirely dependent on a single ICA, and therefore occlusion may be asymptomatic. However, if the circle of Willis does not provide blood flow to the ipsilateral side, the consequence of ICA occlusion is massive hemispheric infarction, with resultant sensorimotor deficits of the contralateral face, arm, and leg (ie, hemiparesis and hemihypesthesia), aphasia when the dominant hemisphere is involved, and contralateral neglect when the nondominant hemisphere is involved. When there is chronic occlusion of the contralateral ICA, then acute occlusion may result in bihemispheric infarction, with resultant coma and quadriparesis.15,16 Acute occlusion of the MCA stem results in sensorimotor deficits of the contralateral face, arm, and, to a lesser degree, leg; homonymous hemianopsia; deviation of the head toward the side of the lesion, ipsilateral gaze preference; global aphasia when the dominant hemisphere is involved; and contralateral neglect (including anosognosia in severe cases) when the nondominant hemisphere is involved.15,16
Acute occlusion of the superior division of the MCA results in dense sensorimotor deficits of the contralateral face, arm, and, to a lesser degree, leg; ipsilateral deviation of the head, ipsilateral gaze preference; and global aphasia (that later improves to Broca’s aphasia) when the dominant hemisphere is involved.15,16
Acute occlusion of the inferior division of the MCA results in superior quadrantanopsia; Wernicke’s aphasia when the dominant hemisphere is involved; and contralateral neglect when the nondominant hemisphere is involved.15,16
Acute occlusion of the ACA distal to the anterior communicating artery results in motor deficits involving the contralateral foot, leg, and, to a lesser degree, upper extremity, with sparing of the hand and face. Acute occlusion of the ACA proximal to the anterior communicating artery is usually well tolerated as a result of collateral blood flow.15,16
Acute occlusion of the large arteries of the posterior circulation results in complex and variable presentations owing to the potential involvement of multiple structures (eg, brainstem, cerebellum, visual cortex), and individual differences in vascular anatomy. Manifestations may include sensorimotor deficits of the ipsilateral face and contralateral extremities, loss of vision of one or both homonymous visual fields, ataxia, vertigo, diplopia or oculomotor deficits (eg, skew deviation), dysphagia, and dysarthria. Non-neurological manifestations may include cardiac dysrhythmias and respiratory dysfunction owing to medullary involvement.15,16,19
What are the underlying causes of acute large artery thrombosis?
872
The most common cause of acute large vessel thrombosis; risk factors include hypertension, diabetes mellitus, smoking, and advancing age. A 38-year-old woman with Ehlers-Danlos syndrome presents with sudden-onset neck and head pain followed by visual deficits and vertigo. A young woman presents with neurologic deficits after an episode of migraine with aura and is found to have associated infarction on neuroimaging. The primary cause of renal artery stenosis in young and middle-aged women. Treatment for this vascular condition usually includes immunosuppressive therapy. Also associated with hemorrhagic stroke, this condition is described by bilateral stenosis of the internal carotid arteries and proximal branches. A 58-year-old man presents with stroke and is found to have a hematocrit of 60%.
What noninvasive studies are available for the evaluation of cerebral artery atherosclerosis?
How often is ischemic stroke caused by arterial dissection?
What are the causes of pathologic cerebral vasoconstriction?
Which arteries are most commonly affected by fibromuscular dysplasia?
Primary angiitis of the central nervous system (PACNS) affects cerebral arteries of what size? What are the demographic differences among adult patients with moyamoya disease who present with hemorrhagic and ischemic strokes? Which blood disorders are associated with thrombotic stroke?
Atherosclerosis.20
Vertebral artery dissection.
Pathologic cerebral vasoconstriction (ie, vasospasm).
Fibromuscular dysplasia.
Vasculitis.
Moyamoya disease.
Polycythemia vera.
Noninvasive imaging options to evaluate for cerebral artery atherosclerosis include transcranial Doppler ultrasonography, CTA, and MRA. CTA is associated with a sensitivity of approximately 97% and specificity of approximately 99.5% for the detection of lesions with >50% stenosis (compared with conventional angiography).20 Cervical artery dissection (ie, carotid or vertebral artery dissection) causes around 20% of ischemic strokes in young and middle-aged patients. Carotid dissection is more common than vertebral dissection.21,22 Causes of pathologic cerebral vasoconstriction include migraine headache, recreational drug use (eg, cocaine), massive subarachnoid hemorrhage, eclampsia, and reversible cerebral vasoconstriction syndrome.15 The renal arteries are most commonly involved in patients with fibromuscular dysplasia (80%) (see Figure 39-2), followed by the extracranial carotid arteries (75%), vertebral arteries (35%), mesenteric arteries (25%), and the intracranial carotid arteries (20%).23 PACNS predominantly affects the medium- and small-sized arteries. Radiographic findings involving the large-sized arteries should raise suspicion for an alternative diagnosis.24 In the United States, adults with moyamoya disease overwhelmingly present with ischemic stroke (approximately 80%). In Asian countries, there is a higher rate of hemorrhagic stroke in adults with moyamoya disease.10
Thrombotic stroke can be related to the following blood disorders: sickle cell anemia, polycythemia vera, thrombocytosis, disseminated intravascular coagulation, protein C or S deficiency, antithrombin III deficiency, antiphospholipid antibody syndrome, heparin-induced thrombocytopenia, and hyperhomocysteinemia. Blood disorders account for up to 5% of all strokes.19
873
Ischemic Stroke Caused by Acute in Situ OCCLUSION OF A Small VESSEL
Which cerebral arteries are classified as small vessels? What is the tempo of a stroke related to small vessel occlusion? What is a lacunar infarction? What are the risk factors for lacunar infarction? How common are lacunar infarctions? In what locations of the brain do lacunar infarctions most often occur?
Small cerebral arteries include the penetrating arteries that come off the vertebral artery, basilar artery, MCA, and the circle of Willis, feeding subcortical structures such as the basal ganglia (eg, putamen, caudate, globus pallidus, thalamus), subcortical white matter (eg, internal capsule, corona radiata), and pons.15
Small vessel strokes tend to develop more quickly than large vessel strokes, but not as suddenly as embolic strokes.15
A lacunar infarction occurs as a result of the occlusion of a small artery (50-200 µm in diameter). When macrophages remove the involved tissue, a small cavity, or lacune, is left behind. These range from 3 to 15 mm in diameter.15 The strongest risk factor for lacunar infarction is chronic hypertension, which is present in the vast majority of patients. Other risk factors include diabetes mellitus and hyperlipidemia.15
In the industrialized world, lacunar strokes account for around one-quarter of all ischemic strokes.25
In descending order of frequency, lacunes occur in the putamen, caudate, thalamus, pons, internal capsule, and deep in the central hemispheral white matter (Figure 43-8).15
FIGURE 43-8 Topographic patterns of cerebral infarction. A, Territorial infarction (from posterior cerebral artery occlusion). B,
What clinical syndromes are associated with lacunar infarctions? How are lacunar infarctions diagnosed? What general medical treatment should be considered for acute lacunar infarction? What is the prognosis of lacunar
Watershed border-zone infarction (between the territories of the anterior cerebral artery and the middle cerebral artery). C, Internal border-zone infarction (deep middle cerebral artery territory). D, Lacunar infarction (lenticulostriate-penetrating artery occlusion). (Adapted from Marshall R, Mayer S. On Call Neurology. 3rd ed. Philadelphia, PA: Saunders; 2007.)
The quality and severity of deficits associated with lacunar infarctions are dependent on location. The most common syndromes include, in descending order of frequency, pure motor hemiparesis (eg, when the posterior limb of the internal capsule is involved), pure hemisensory stroke (eg, when the thalamus is involved), ataxic hemiparesis (eg, when the pons, midbrain, internal capsule, or parietal white matter is involved), and clumsy hand dysarthria syndrome (eg, when the midpons is involved). Multiple lacunes deep in both hemispheres can result in gait disorders and dementia. In virtually all forms of lacunar infarction, there is an absence of cortical deficits (eg, seizure, aphasia, amnesia, agnosia, apraxia, dysgraphia, alexia), which can be a clue to the diagnosis.15,26 Unlike occlusions of large arteries, involvement of the smaller arteries cannot be visualized on angiography. Diagnosis is based on the clinical syndrome and the presence of characteristic noncortical infarctions on head imaging, which are more likely to be found on MRI than CT.15 There is some evidence that administering intravenous thrombolytics in select patients with acute lacunar infarction may be beneficial, using the same eligibility criteria as in patients with large vessel ischemic stroke. However, more data is needed in this subgroup of ischemic stroke. Aspirin should be given within 24 to 48 hours after stroke (but not within 24 hours of fibrinolytic therapy if it is given).25
In general, short-term recovery is more favorable in patients with lacunar infarction compared with other causes of ischemic stroke, sometimes with complete recovery even after a severe event. However, many patients experience some degree of
874
infarction? long-term sequelae.15
What are the underlying causes of acute small vessel in situ occlusion?
This abnormal vascular process occurs within the small arteries and is associated with chronic hypertension and diabetes mellitus. This vasculopathy is the leading cause of large vessel thrombotic stroke.
What is the mechanism of small vessel occlusion related to lipohyalinosis?
What is the mechanism of small vessel occlusion related to atherosclerosis?
Lipohyalinosis.
Atherosclerosis.
Lipohyalinosis, which is a vascular lesion characterized pathologically by the presence of fibrinoid material and lipid deposition within the arterial wall with associated medial hypertrophy, leads to encroachment on the arterial lumen with eventual obliteration.27 The origins of the small arteries can become blocked by atheromata found within the parent arteries, resulting in occlusion.27
875
Embolic Stroke
What is the tempo of stroke related to Embolic strokes tend to occur suddenly with maximal deficits at onset. Some patients with embolism? chronic significant cerebral atherosclerotic disease develop robust collateral circulation, mitigating the effects of an acute embolic event.15,28 What pattern of infarction can be a clue In contrast to thrombotic strokes, which typically present in a single vascular territory, embolic to the presence of an embolic source of strokes can involve multiple cerebral vascular territories, as well as noncerebral vascular ischemic stroke? territories (eg, kidney).
What are the general mechanisms of embolic stroke?
A large proportion of cardiac output goes to the brain. A gift from one artery to another. Venous thromboembolism.
How common is cardioembolic stroke? What are the major risk factors for cardioembolic stroke? What are the most common sites of atherosclerotic lesions that give rise to artery-to--artery embolic stroke? What features of atherosclerotic disease confer Increased risk for embolic stroke?
What is the cornerstone of medical therapy for patients at risk for artery-to-artery embolism? What clinical syndromes are associated with emboli from the internal carotid arteries? What clinical scenario is described by the expression “spectacular shrinking
Cardioembolism.
Artery-to-artery embolism. Paradoxical embolism.
Cardioembolic stroke is the most common underlying cause of embolic stroke and accounts for one-quarter of all strokes in the general population of the industrialized world.19 Risk factors for cardioembolic stroke include atrial fibrillation, mitral valve stenosis, prosthetic cardiac valve, recent myocardial infarction, left ventricular or atrial thrombus, infectious and non-infectious endocarditis, dilated cardiomyopathy, and intracardiac tumors (eg, atrial myxoma).19
The aortic arch is a common source of atheroembolic stroke. In the anterior circulation, the most common sites include the carotid artery (most commonly at the carotid artery bifurcation and siphon) and the M1 segment of the MCA. In the posterior circulation, the most common sites include the first and fourth segments of the vertebral artery and the first segment of the basilar artery.29
For atherosclerotic disease of the aorta, the size of the plaques correlates with risk of ischemic stroke (plaques >4 mm in thickness confer highest risk). Transesophageal echocardiography allows for the detection and measurement of aortic arch disease. For the cerebral arteries, the size of the plaque also correlates with risk of ischemic stroke (there is Increased risk with lesions causing >50% stenosis and an even greater risk with >70% stenosis). Noninvasive imaging modalities used to assess the burden of cerebral atherosclerotic disease include transcranial Doppler ultrasonography, MRA, and CTA. Conventional angiography may be necessary in some patients.28,29 Antiplatelet therapy (either mono or dual depending on clinical factors) is the cornerstone of medical therapy used to prevent ischemic stroke in patients at risk for artery-to-artery embolism.28
The clinical manifestations of ICA-related emboli are variable owing to individual disparity in collateral flow that develops as a result of long-standing ICA disease, ranging from asymptomatic to devastating hemispheric infarction.16
“Spectacular shrinking deficit” is an expression used to describe the clinical manifestations of a migrating embolus originating from the ICA. The initial embolus lodges in the stem of the MCA, resulting in a major hemispheric deficit, but the deficit improves as the embolus moves to one of the smaller branches of the MCA.16
876
deficit”? What coexistent conditions must be present for paradoxical embolism to occur? What is the prevalence of patent foramen ovale in the general population?
Paradoxical embolism, which occurs when embolic material from the venous circulation transfers to the arterial circulation, requires both venous emboli and a right-to-left shunt (eg, patent foramen ovale [PFO], atrial septal defect, or ventricular septal defect). Right-to-left shunting can occur in patients with left-to-right intracardiac shunt when right-sided pressures are transiently Increased (eg, during Valsalva).30
PFO occurs in one-quarter of the general population. It is present in one-half of patients with cryptogenic stroke.31
877
Watershed Stroke
How common is watershed infarction? Where are the watershed areas of the brain?
What are the 2 main clinical manifestations of cortical watershed infarction?
What are the clinical manifestations of subcortical watershed infarction?
Based on autopsy studies, watershed infarctions account for approximately 10% of all strokes. This may be an underestimation because watershed infarction is rarely fatal.32 The cortical (external) watershed zones include the anterior region within the frontal and parietal parasagittal cortex (MCA/ACA territory) and the posterior region within the parieto-temporo-occipital cortex (MCA/PCA). The incidences of anterior and posterior watershed infarctions are similar. The subcortical (internal) watershed zones include the corona radiata (deep/superficial perforators of the MCA) and the centrum semiovale (superficial perforators of the ACA/MCA) (see Figure 43-8).16,32 Anterior watershed ischemia (MCA/ACA territory) results in a (proximal greater than distal) sensorimotor deficit of the upper extremity with possible involvement of the lower extremity and sparing of the face and hand (described as “man-in-the-barrel” syndrome when the infarction is bilateral). Posterior watershed ischemia (MCA/PCA territory) results in visual agnosias, various degrees of visual field deficits (eg, hemianopsia, quadrantanopia, cortical blindness), transcortical aphasia or contralateral neglect (depending on involvement of the dominant or nondominant hemisphere), and, when the infarction is bilateral, the Balint syndrome (ie, simultagnosia, optic ataxia, and oculomotor apraxia). Rarely, both syndromes may occur together. Other features of watershed infarction include syncope at onset and focal limb shaking.15,16 Confluent subcortical watershed infarction may result in contralateral sensorimotor deficits of the arm, leg, and face, and focal cognitive and behavioral dysfunction (with poor recovery), whereas partial subcortical watershed infarction may result in brachiofacial sensorimotor deficits and focal cognitive and behavioral dysfunction (with good recovery).33
What are the causes of watershed stroke?
A 65-year-old man with elevated jugular venous pressure, an S3 gallop, pulsus alternans, and cold extremities. Global cerebral hypoperfusion with or without systemic hypotension.
Cardiogenic shock.
Diffuse atherosclerosis of the cerebral arteries.
Is systemic hypotension more likely to result in Systemic hypotension is associated with both types of watershed infarctions but cortical or subcortical watershed stroke? most often results in subcortical watershed stroke.32 Which cerebral artery is most commonly associated The ICA, being the most proximal cerebral artery, is most likely to cause watershed with watershed stroke as a result of atherosclerotic stroke when there is severe atherosclerotic disease. The condition can be unilateral disease? or bilateral.16,32 It should be recognized that cerebral hypoxia caused by entities such as hypoxemia and carbon monoxide poisoning (although not traditionally considered to be causes of stroke) could result in brain tissue infarction.
878 Case Summary A 48-year-old woman on combined oral contraceptive pill presents focal neurologic deficits and is found to have lower extremity deep with sudden-onset right flank pain followed by the development of vein thrombosis, a lack of contrast enhancement of the right kidney on imaging,tand evidence oflan ischemic stroke involving the distribution What is the most likely cause of stroke in this patient? Paradoxical embolism. of the lef middle cerebra artery.
879
likely explained by an embolic event to the right renal artery with resultant acute obstruction and ischemic injury. Bonus Questions
pain, hematuria, and lack of contrast enhancement Which features of this case make embolic stroke What is the most likely source of paradoxical What is the most likely explanation for the flank of the right kidney in this case? likely? embolism in this case?
What acute medical treatment should be considered for the patient in this case?
What is the role of mechanical thrombectomy in patients with ischemic stroke?
investigate the paradoxical nature of the stroke? What study should be performed in this case to If a patent foramen ovale is discovered on echocardiography in this case, what treatment strategies should be used for secondary prevention of stroke?
The constellation of flank pain, hematuria, and lack of right kidney enhancement on imaging in this case (see Figure 43-1) is most
The neurologic deficits in this case occurred suddenly and were maximal at onset, characteristic of the tempo of embolic stroke. Involvement of multiple tissue beds in the body is also suggestive of embolic phenomena. The deep vein thrombosis of the left leg is the likely source of emboli in this case. In order for paradoxical embolism o occur, there must also be a coexistent right-to-left shunt; a patent foramen ovale is statistically most likely, present in one-quartertof the general population. 31 Given that the onset of the stroke is within 3 hours and there aretno contraindications (eg, active internal bleeding), the patient in e In the setting of proximal large artery occlusion of the anterior circulation, such as in this case, mechanical thrombectomy is this case should be treated with an intravenous fibrinolytic agen (eg, tPA). Aspirin should be given within 24 to 48 hours of strok (but not within 24 hours of intravenous thrombolytic administration). beneficial in patients who do and do not receive intravenous thrombolytics, and should be pursued unless there are contraindications. Proximal large artery occlusions of the posterior circulation may also benefit from mechanical thrombectomy.17 A transthoracic echocardiogram with agitated saline contrast should be performed to evaluate for intracardiac shunt.
foramen ovale. In some cases, such as when there are recurrent paradoxical emboli despite appropriate medical therapy, it may be Antiplatelet or systemic anticoagulants are recommended for patients with cryptogenic stroke who are found to have a patent reasonable to consider mechanical closure of the PFO. 34
880 Key Points
Stroke occurs when there is acute infarction of brain tissue Others include diabetes mellitus, atrialffibrillation, dyslipidemia,. Strokes can be hemorrhagic (20%) or ischemic (80%). resulting in neurologic deficits. Chronic hypertension is the most signi icant risk factor for stroke smoking, and family history. Hemorrhagic stroke causes injury due to mechanical compression of brain tissue and local toxicity from blood breakdown products; ischemic stroke causes injury due to insufficient oxygen and nutrient delivery to brain tissue.
Neuroimagingtis necessary to distinguish between hemorrhagic compared with ischemic stroke, but long-term functional status and ischemic s roke. Hemorrhagic stroke is associated with higher 30-day mortality among survivors is similar between the two. Hemorrhagic stroke can occur as a result of intracerebral
hemorrhage or SAH, each with different underlying causes and Ischemic stroke can occur as a result of acute in situ occlusion infarction, each with different underlying causes and treatments. treatments. (including large vessel and small vessel), embolism, or watershed There are well-described large vessel and small vessel ischemic stroke syndromes depending on the distribution of infarction.
Involvement of noncontiguous vascular territories is suggestive of Eligible patients with acute ischemic,strokeirelatedlto thrombosis embolic stroke. Embolic stroke can occur as a result of card oembo ism (most common), artery-to-artery embolism or paradoxical embolism. or embolism may benefit from intravenous tPA if given within a certain time from confirmed or estimated stroke onset.
Mechanical thrombectomy may be beneficial in select patients with Watershed stroke occurs when a systemic process results in global brain located between neighboring vascular territories, and is best proximal large artery occlusion. cerebral hypoperfusion that affects the vulnerable regions of the managed by addressing the underlying systemic process.
881
882
References 1. Prabhakaran S, Ruff I, Bernstein RA. Acute stroke intervention: a systematic review. JAMA. 2015;313(14):1451-1462.
- Kernan WN, Ovbiagele B, Black HR, et al. Guidelines for the prevention of stroke in professionals from the American Heart Association/American Stroke Association. patients with stroke and transient ischemic attack: a guideline for healthcare Stroke. 2014;45(7):2160-2236. Benjamin EJ, Blaha M , Chiuve SE, et al. Heart disease and stroke statistics-2017
-
update: a report fromJthe American Heart Association. Circulation. 2017; 135(10):e146- e603.
-
van der Worp HB, van Gijn J. Clinical practice. Acute ischemic stroke. N Engl J Med. 2007;357(6):572-579.
-
Smith SD, Eskey CJ. Hemorrhagic stroke. Radiol Clin North Am. 2011;49(1):27-45.
-
Ariesen MJ, Claus SP, Rinkel GJ, Algra A. Risk factors for intracerebral hemorrhage in the general population: a systematic review. Stroke. 2003;34(8):2060-2065. Sturgeon JD, Folsom AR, Longstreth WT Jr, Shahar E Rosamond WD, Cushman
-
Risk factors for intracerebral hemorrhage in a pooled,prospective study. Stroke. M. 2007;38(10):2718-2725.
-
Hemphill JC III, Greenberg SM, Anderson CS, et al. Guidelines for the management of the American Heart Association/American Stroke Association. Stroke. 2015;46(7):2032- spontaneous intracerebral hemorrhage: a guideline for healthcare professionals from 2060.
- Perel P, Roberts I, Bouamra O, Woodford M, Mooney J, Lecky F. Intracranial bleeding in patients with traumatic brain injury: a prognostic study. BMC Emerg Med.
2009;9:15. 10. Scott RM, Smith ER. Moyamoya disease and moyamoya syndrome. N Engl J Med. 2009;360(12):1226-1237. . Vlak MH, Rinkel GJ, Greebe P, Greving JP, Algra A. Lifetime risks for aneurysm
11subarachnoid haemorrhage: multivariable risk stratification. J Neurol Neurosurgal Psychiatry. 2013;84(6):619-623.
-
Polmear A. Sentinel headaches in aneurysmal subarachnoid haemorrhage: what is the true incidence? A systematic review. Cephalalgia. 2003;23(10):935-941.
-
Connolly ES Jr, Rabinstein AA, Carhuapoma JR, et al. Guidelines for the management the American Heart Association/American Stroke Association. Stroke. 2012;43(6):1711- of aneurysmal subarachnoid hemorrhage: a guideline for healthcare professionals from 1737. . Armin SS, Colohan AR, Zhang JH. Traumatic subarachnoid hemorrhage: our current 14understanding and its evolution over the past half century. Neurol Res. 2006;28(4):445- 452.
-
Ropper AH, SamuelsiMA, Klein JP, eds. Adam and Victor’s Principles of Neurology. 10th 16. Gavrilescu T, Kase CS. Clinical stroke syndromes: clinical-anatomical correlations. ed. China: McGraw-H ll Education; 2014. Cerebrovasc Brain Metab Rev. 1995;7(3):218-239.
-
Goyal M, MenontBK, van Zwam WH, et al. Endovascular thrombectomy after large- vessel ischaemic s roke: a meta-analysis of individual patient data from five randomised trials. Lancet. 2016;387(10029):1723-1731.
883
- Jauch EC, Saver JL, Adams HP Jr, et al. Guidelines for the early management of patients with acute ischemic stroke: a guideline for healthcare professionals from the
American Heart Association/American Stroke Association. Stroke. 2013;44(3):870-947. 19Evaluation and,management of transient ischemic attackland minor cerebral infarction. . Flemming KD Brown RD Jr, Petty GW, Huston J III, Ka lmes DF, Piepgras DG. Mayo Clin Proc. 2004;79(8):1071-1086.
-
Banerjee C, Chimowitz MI. Stroke caused by atherosclerosis of the major intracranial arteries. Circ Res. 2017;120(3):502-513.
-
Dodick D. Headache as a symptom of ominous disease. What are the warning signals? Postgrad Med. 1997;101(5):46-50, 5-6, 62-4.
-
Griffiths D, Sturm J. Epidemiology and etiology of young stroke. Stroke Res Treat. 2011;2011:209370.
-
Varennes L, Tahon F, Kastler A, et al. Fibromuscular dysplasia: what the radiologist should know: a pictorial review. Insights Imaging. 2015;6(3):295-307.
-
Berlit P.:Diagnosis and treatment of cerebral vasculitis. Ther Adv Neurol Disord. 2010;3(1) 29-42.
-
Behrouz R, Malek AR, Torbey MT. Small vessel cerebrovascular disease: the past, present, and future. Stroke Res Treat. 2012;2012: 839151.
-
Arboix A, Marti-Vilalta JL, Garcia JH. Clinical study of 227 patients with lacunar infarcts. Stroke. 1990;21(6):842-847.
-
Caplan LR. Lacunar infarction and small vessel disease: pathology and pathophysiology. J Stroke. 2015;17(1):2-6. . Holmstedt CA, Turan TN, Chimowitz MI. Atherosclerotic intracranial arterial
28stenosis: risk factors, diagnosis, and treatment. Lancet Neurol. 2013;12(11):1106-1114. 29Conference. IV. Prevention and rehabilitation of stroke. Etiology of stroke. Stroke. . Mohr JP, Albers GW, Amarenco P, et al. American Heart Association Prevention 1997;28(7):1501-1506.
- Pittenger B, Young JW, Mansoor AM. Subretinal abscess. BMJ Case Rep. 2017;2017. 31. Saver JL. Clinical practice. Cryptogenic stroke. N Engl J Med. 2016;374(21):2065-2074. 32carotidiartery disease: review of cerebral perfusion studies. Stroke. 2005;36(3):567-577. 33. Bladin CF,iChambers BR. Clinical features, pathogenesis, and computed tomographic 34. Nayor M, Maron BA. Contemporary approach to paradoxical embolism. Circulation. . Momj an-Mayor I, Baron JC. The pathophysiology of watershed infarction in internal characterist cs of internal watershed infarction. Stroke. 1993;24(12):1925-1932. 2014;129(18):1892-1897.
884
CHAPTER 44
885
Weakness
886 Case: A 52-year-old man with a “waddling” gait A previously healthy 52-year-old man presents to the clinic complaining of weakness. Several weeks ago he developed soreness
and heaviness of the legs, which forced him to stop his daily exercise difficulty rising from a seated position. He additionally describes routine. The symptoms progressed, and he soon began having recent voice changes and swallowing difficulties. The symptoms do not change over the course of the day. He does not describe any ocular Cranial.nerves are intact. Mild atrophy of the proximal muscles is . difficulty. Sensation is intact. Reflexes are normal and symmetric. symptoms. Vital signs are unremarkable. The patient has a “waddling” gait present Strength of the deltoids and hip flexors are graded 3/5; handgrip strength is 5/5; and toe raises are performed without There is discoloration around the eyes (Figure 44-1). There are violaceous and scaly papules over the dorsum of the
metacarpophalangeal and interphalangeal joints (Figure 44-2).
FIGURE 44-1 (From Schalock PC, Hsu JTS, Arndt KA. Lippincott’s Primary Care Dermatology. Philadelphia, PA: Wolters Kluwer Health; 2011.)
FIGURE 44-2 (Courtesy of Peter D. Sullivan, MD.)
What is the most likely cause of weakness in this patient?
What is muscle weakness? What is a motor unit?
Weakness is a reduction in the power that can be generated by muscle.1 A motor unit consists of a nerve cell, the axon from that nerve cell, and the muscle fibers innervated by the terminal branches of that axon (Figure 44-3).2
887
FIGURE 44-3 A motor unit is made up of a motor neuron cell body (which resides in the gray matter of the spinal cord)
What is fatigability?
What is asthenia?
What is bradykinesia? What is muscle bulk? What is muscle tone?
What is muscle fibrillation? What is muscle fasciculation? What is clonus?
What is paralysis?
What is the meaning of the term “plegia”? What is the meaning of the term “paresis”? What scale can be used to grade muscle strength?
How can true involuntary muscle weakness be distinguished from voluntary weakness during resistance testing? What are the 4 anatomic categories of weakness?
along with its axon (which exits the spinal cord) and the muscle fibers innervated by the terminal branches of the axon. (From Ives JC, Motor Behavior: Connecting Mind and Body for Optimal Performance, 2nd ed. Philadelphia, PA: Wolters Kluwer; 2018.)
Fatigability describes a gradual reduction in power with repetitive use of a muscle. It can be a feature of neuromuscular disease, such as myasthenia gravis. Fatigability should not be confused with fatigue such as that experienced by patients with systemic conditions (eg, anemia), in which true weakness is not present.2 Asthenia, often confused with weakness, is characterized by feelings of weariness, exhaustion, and disinclination to engage in and sustain physical activity, in the absence of true muscle weakness. Similar terms include fatigue, lassitude, and lethargy.2 Bradykinesia, sometimes misinterpreted as weakness, is an abnormal increase in the time required to initiate and complete movement.1 Bulk refers to overall muscle mass. Atrophy, which can be symmetric or asymmetric, occurs with certain causes of muscle weakness, particularly lower motor neuron (LMN) processes. Tone refers to the involuntary resistance of muscle to passive stretch. Spasticity is a type of velocity-dependent (ie, more obvious with fast movements) hypertonicity that predominantly affects the antigravity muscles (ie, upper limb flexors and lower limb extensors), and is seen with upper motor neuron (UMN) disorders. Rigidity is a type of velocity-independent hypertonicity that affects flexors and extensors equally, and is seen with extrapyramidal disorders such as Parkinson’s disease. Flaccidity describes a decrease in tone, and is seen with LMN disorders (it can also be seen in the acute phase of UMN disorders [eg, spinal shock]).1 Fibrillation refers to involuntary contraction of individual muscle fibers as a result of denervation. It is too fine to be visualized with the naked eye but can be observed on electromyography.2 Fasciculation refers to involuntary contraction of 1 or more motor units resulting in a visible twitch of a muscle fascicle. It can occur as a result of Increased neuromuscular irritability associated with lower motor neuron disease.2 Clonus refers to sustained and rhythmic involuntary muscular contractions that occur with a frequency of 5 to 7 Hz in response to a stretch reflex. It is a manifestation of the hyperreflexic state of spasticity and may be an indication of upper motor neuron disease.2 Paralysis refers to the complete loss of voluntary movement related to interruption of the motor pathway anywhere from the cerebral cortex to the muscle fiber.2 The term plegia denotes severe weakness or paralysis. Monoplegia refers to paralysis of 1 limb; hemiplegia refers to paralysis of 1 side of the body (involving the arm, the leg, and sometimes the face); paraplegia refers to paralysis of both legs; and quadriplegia refers to paralysis of all 4 extremities.2 The term paresis denotes partial loss of motor function.2
The following 0-5 scale is commonly used for grading muscle weakness2: 0—Complete paralysis 1—Minimal contraction 2—Active movement when gravity is eliminated 3—Full movement against gravity but cannot provide resistance against manual muscle opposition 4—Active movement against gravity and resistance but overcome by manual muscle opposition 5—Normal strength True weakness gives way smoothly to resistance. In voluntary release, the patient may successfully resist for a few moments before suddenly letting go (ie, “giveaway” weakness). In other cases, the patient may attempt to mimic the gradual release of true weakness, but this produces a series of small phases akin to a “cogwheel” (not to be confused with true cogwheeling in basal ganglia disorders). In some circumstances, voluntary release does not rule out the presence of true weakness (eg, concurrent pain may lead to voluntary release in those with true weakness).3
Weakness can be caused by disorders of the UMN, LMN, neuromuscular junction (NMJ), or muscle (Figure 44-4).
888
FIGURE 44-4 Relationship between the upper motor neuron, lower motor neuron, neuromuscular junction, and muscle. (Adapted From Drislane FW, Acosta J, Caplan L, Chang B, Tarulli;A. Blueprints Neurology, 4rd ed. Philadelphia, PA: Wolters Kluwer Health 2013.)
What are the general characteristics of the 4 categories of weakness?
UMN lesions are proximal to the anterior horn cell and are characterized by Increased tone (spasticity), hyperreflexia, and minimal atrophy; proximal muscles are typically affected more than distal muscles. LMN lesions involve the structures distal to and including the anterior horn cell and are characterized by flaccid tone, diminished reflexes, fasciculations, and atrophy; distal muscles are typically affected more than proximal muscles. NMJ disorders present with variable distributions of weakness (classically proximal >distal). Myopathic weakness tends to be symmetric and most pronounced in the proximal muscles.1
What are the effects of upper motor neuron lesions, lower motor neuron lesions, and myopathies on muscle bulk, fasciculations, tone, tendon reflexes, and Babinski reflex?
What finding on physical examination is suggestive of a disorder of the neuromuscular junction?
Sign UMN LMN Myopathy Atrophy None/minimal Severe Mild Fasciculations None Common None Tone Increased Decreased Normal/Decreased (spastic) Reflexes Increased Decreased Normal/Decreased Babinski Present Absent Absent *Reflexes and tone may be Decreased in UMN lesions in the period immediately following the injury (eg, spinal shock). Fasciculations are most often seen in LMN disorders involving the anterior horn cell and/or nerve root.1 The presence of fatigable weakness is suggestive of disorders that affect the NMJ, particularly postsynaptic disorders (eg, myasthenia gravis).
889
Weakness Related to Upper Motor Neuron Lesions
What is an upper motor neuron? Upper motor neuron lesions occur within which 2 general anatomic structures?
What direct connections between the brain and spinal cord control voluntary movement?
In which general regions of the brain do upper motor neuron lesions occur? What are the general characteristics of the weakness caused by upper motor neuron lesions?
What are the characteristics of the facial weakness caused by lesions of the corticobulbar tracts? Restricted weakness (eg, hand and arm or foot and leg) can result from upper motor neuron lesions involving which general regions of the brain? Hemiparesis can result from upper motor neuron
UMNs are neurons originating in the motor cortex that synapse with LMNs of the brainstem (brainstem nuclei) and spinal cord (anterior horn cells), which in turn innervate effector organs (eg, muscle) (see Figure 44-4). UMN lesions can involve the brain or spinal cord (see Figure 44-4).
The cell bodies of the UMNs (gray matter) reside mainly in the motor cortex. The axons from these neurons (white matter) form the corticobulbar and corticospinal tracts, which travel through the subcortical white matter and internal capsule to the brainstem. The corticobulbar tracts innervate the LMNs of the brainstem, whereas the corticospinal tracts continue on to innervate the LMNs of the spinal cord (Figure 44-5). The indirect pathways between the brain and spinal cord include the rubrospinal, reticulospinal, vestibulospinal, and tectospinal (extrapyramidal) tracts.1,2
FIGURE 44-5 Motor pathways: corticospinal and corticobulbar tracts. (From Hogan-Quigley B, Palm ML, Bickley LS. Bates’ Nursing Guide to Physical.Examination and History Taking, 2nd ed. Philadelphia, PA: Wolters Kluwer; 2017 )
Within the brain, UMN lesions can occur in the cerebral cortex, subcortical white matter, internal capsule, and brainstem.1 UMN lesions always affect a group of muscles (never an individual muscle), with proximal >distal involvement, disproportionately affecting the upper limb extensors (causing pronator drift) and lower limb flexors. Facial movements that are bilateral (eg, eyes, jaw) are spared or affected to a small degree because of bilateral innervation (Broadbent’s law). Deficits in fine motor movement (eg, finger tapping) may also be observed.2 Lesions of the corticobulbar tracts usually involve the contralateral muscles of the lower face and tongue while there is sparing of the muscles that habitually produce bilateral movement, including extraocular, upper facial (eg, forehead), pharyngeal, and jaw muscles. This occurs as a result of the bilateral innervation of those muscles.1,2 Because of the somatotopic organization of the corticospinal system, a discrete lesion of the cortex or subcortex could result in limited zones of weakness (see Figure 44-5).2
Hemiparesis, the most frequent form of paralysis, can occur as a result of lesions of the corticospinal tracts above the midcervical spinal cord.1,2
890
lesions involving which general regions of the brain or spinal cord? What additional neurologic manifestations often occur along with hemiparesis in patients with lesions of the cerebral cortex? Pure motor hemiparesis of the face, arm, and leg can result from upper motor neuron lesions involving which regions of the brain? Ipsilateral cranial nerve signs with contralateral hemiparesis can result from upper motor neuron lesions involving which region of the brain? Hemiparesis in the absence of cranial nerve signs or facial weakness is suggestive of upper motor neuron lesions involving which region of the spinal cord? Paraparesis can result from upper motor neuron lesions involving which region of the spinal cord? Quadriparesis can result from upper motor neuron lesions involving which region of the spinal cord?
Lesions of the cerebral cortex often result in disorders of language and visual-spatial integration (eg, neglect), and can cause cortical sensory disturbances (eg, agraphesthesia), apraxia, and seizure. The presence of any of these findings in addition to hemiparesis is suggestive of a lesion within the cerebral cortex.1 Because the descending motor fibers from the cortex converge and collect within the posterior limb of the internal capsule, cerebral peduncle, and upper pons, even small lesions in these regions can result in pure motor hemiparesis, where the face, arm, hand, leg, and foot are affected to a similar degree.1
Lesions of the brainstem can result in ipsilateral cranial nerve signs with contralateral hemiparesis.1
Lesions of the high cervical spinal cord can result in ipsilateral hemiparesis in the absence of cranial nerve signs or facial weakness.1
Lesions at or below the thoracic spinal cord that involve both corticospinal tracts can result in paraparesis.1
Like hemiparesis, quadriparesis can occur as a result of lesions of the corticospinal tracts above the midcervical spinal cord; however, in the case of quadriparesis, both corticospinal tracts must be involved.
891
Weakness Related to Upper Motor Neuron Lesions of the Brain
What are the causes of Weakness Related to the brain?
Risk factors for this condition include hypertension, smoking, hyperlipidemia, and atrial fibrillation. A 67-year-old man with non–small cell lung cancer presents with insidious-onset weakness of the left arm with Increased muscle tone and tendon reflexes. Fever, papilledema, and focal neurologic deficits in a patient with recent otitis media. A young woman with optic neuritis. An acute demyelinating condition that typically follows a viral infection.
Stroke. Metastatic brain tumor.
Brain abscess. Multiple sclerosis (MS). Acute disseminated encephalomyelitis (ADEM).
Which side of the body would be affected by a stroke Lesions involving the right cerebral cortex, cerebral white matter (ie, corona involving the right cerebral cortex, cerebral white radiata), or internal capsule result in hemiparesis of the contralateral (left) side. matter, or internal capsule? What proportion of metastatic brain tumors involve Metastatic lesions predominate in the cerebral hemispheres (80%) compared with the cerebral hemispheres (where the motor cortex is the posterior fossa (20%), reflecting the relative size and blood flow of those located) compared with the posterior fossa (where regions; brain metastases typically involve the gray-white junction.2 the brainstem is located)? What are the 3 general mechanisms of brain abscess Brain abscesses occur as a result of direct extension from extracranial infection (eg, development? sinusitis), hematogenous spread, or direct inoculation following head injury or neurosurgery. What are the manifestations of brainstem Diplopia is the most common manifestation of brainstem involvement in MS; involvement in patients with multiple sclerosis? others include facial sensory symptoms, unstable gait, vertigo, oscillopsia, facial weakness, nausea or vomiting, trigeminal neuralgia, dysarthria, hypoacusia, dysgeusia, somnolence, and dysphagia.4 How is the evaluation of cerebrospinal fluid (CSF) Although CSF is abnormal in most patients with ADEM (characterized by helpful in distinguishing multiple sclerosis from moderate pleocytosis and elevated protein), the presence of oligoclonal bands acute disseminated encephalomyelitis? occurs in only a minority and is more suggestive of MS.5
892
Weakness Related to Upper Motor Neuron Lesions of the Spinal Cord
What are the origins and routes of the corticospinal tracts within the spinal cord? How are the corticospinal tracts organized?
Which arteries supply the spinal cord? What would be the expected result of complete spinal cord transection? Cessation of respiration would result from complete transection of what level of the spinal cord? What are the characteristics of anterior cord syndrome? What are the characteristics of posterior cord syndrome? What is Brown- Séquard syndrome?
Is spinal cord disease associated with upper motor neuron involvement only?
The axons of the UMNs descend through the brainstem; at the cervicomedullary junction, most axons (70%-90%) decussate (cross) into the contralateral corticospinal tract of the lateral spinal cord while a smaller proportion remain ipsilateral in the anterior spinal cord. The axons continue on to innervate the LMNs of the spinal cord (see Figure 44-5).1 The corticospinal tracts are organized somatotopically. The axons that control upper extremity movement are positioned medially, whereas those controlling lower extremity movement are positioned laterally (see Figure 44-5). Central cord syndrome, for example, results in loss of motor function predominantly in the upper extremities with relative sparing of the lower extremities.2 The anterior spinal artery supplies the ventral two-thirds of the spinal cord, whereas the posterior spinal arteries supply the dorsal one-third.2 Spinal cord transection results in interruption of all ascending and descending tracts below the level of the lesion, leading to bilateral complete loss of sensation, motor, and autonomic function. Spinal shock may result from acute cord transection, which is characterized by a temporary state of flaccid paralysis, loss of sensation below the level of the lesion, atonic paralysis of the bladder and bowel, gastric atony, and diminished reflexes.2 Transection of the spinal cord above C3 results in complete cessation of respiration and is commonly fatal.2
Anterior cord syndrome refers to damage to the ventral two-thirds of the spinal cord (supplied by the anterior spinal artery), which contains the corticospinal tracts (resulting in motor deficits below the level of the lesion) and spinothalamic tracts (resulting in loss of pain and temperature sensation below the level of the lesion). There is preservation of fine touch, vibration, and proprioception (carried by the dorsal columns).2 Posterior cord syndrome is characterized by damage to the dorsal columns (fine touch, vibration, and proprioception), resulting in manifestations such as gait (sensory) ataxia and paresthesias. There is preservation of motor function, pain, and temperature (carried by the corticospinal and spinothalamic tracts). Posterior cord syndrome related to vascular disease is much less common than anterior cord syndrome.2 Brown-Séquard syndrome occurs as a result of damage to a single lateral side of the spinal cord. Involved structures include the unilateral corticospinal tract, dorsal column, and spinothalamic tract. Clinical sequelae include ipsilateral weakness, ipsilateral loss of proprioception and vibration, and contralateral loss of pain and temperature.2 Most spinal cord diseases (ie, myelopathies) produce UMN lesions via involvement of the corticospinal tracts; however, LMN signs may be present at the level of the spinal cord lesion when there is local involvement of the anterior horn cell.2
What are the causes of Weakness Related to the spinal cord?
Mass effect. A 44-year-old man becomes quadriplegic after falling off a horse. Fever and paraparesis. Damage to the anterior spinal artery, which supplies the area of the spinal cord housing the corticospinal tracts. A 34-year-old woman with anorexia nervosa slowly develops angular cheilitis, paresthesias, spastic paraparesis, brisk reflexes, and a positive Romberg sign. A 28-year-old woman develops pain with left eye movement and acute-onset paraparesis associated with a white matter lesion of the cervical spine on magnetic resonance imaging (MRI), followed by resolution of symptoms over the next few weeks. Development of myelopathy following immunization. This entity, which is on a
Spinal cord compression (eg, from disc herniation). Trauma.
Infection (eg, epidural abscess). Vascular disease (eg, vasculitis).
Vitamin B12 deficiency.
Multiple sclerosis.
Acute disseminated encephalomyelitis.
Transverse myelitis.
893
disorders such as multiple sclerosis and acute disseminated encephalomyelitis, can be idiopathic or secondary to other conditions (usually autoimmune diseases).
A noninfectious granulomatous disease. A neoplastic process without evidence of spinal cord compression or direct cord involvement. Palmar erythema, spider angiomas, and caput medusae. These myelopathies run in families. Myelopathy on vacation in Hawaii. Scuba divers are at risk.
What are the compressive causes of myelopathy? What are the traumatic causes of myelopathy?
What are the infectious causes of myelopathy?
What are the vascular causes of myelopathy? Which nutritional deficiencies can result in myelopathy? What are the patterns of spinal cord involvement in multiple sclerosis? What is the prognosis of acute disseminated encephalomyelitis?
What is the prognosis of transverse myelitis?
What is the typical cerebrospinal fluid profile in patients with sarcoid myelopathy?
Which laboratory test can be helpful in diagnosing paraneoplastic syndromes of the central nervous system?
Sarcoidosis. Paraneoplastic syndrome.
Hepatic myelopathy. Hereditary myelopathy (eg, hereditary spastic paraparesis). Surfer’s myelopathy. Caisson’s disease (ie, decompression sickness).
Compressive causes of myelopathy include degenerative disease (eg, osteoarthritis), spondylolisthesis, spinal stenosis, disc herniation, tumor (benign or malignant), syringomyelia, epidural abscess, and hematoma. Motor vehicle accidents account for about one-half of all traumatic causes of myelopathy; other causes include falls, inflicted injuries (eg, gunshots), sports-related injuries, radiation treatment, and electrical injury.6 Infectious causes of myelopathy include epidural abscess, acute viral myelitis, human immunodeficiency virus (HIV), tuberculosis, syphilis, fungi (eg, Blastomyces dermatitidis), and parasites (eg, Schistosoma mansoni).7 Vascular causes of myelopathy include thrombi, emboli, vasculitis, hematoma, and vascular malformation (eg, dural arteriovenous fistula).2 Deficiencies of vitamin B12 or copper can result in myelopathy, often presenting as myeloneuropathy (the combination of spinal cord and peripheral nerve involvement).2 The vast majority of patients with MS develop spinal cord lesions, which can be helpful in making the diagnosis when identified with MRI. Most lesions are focal, usually involving the cervical cord although diffuse abnormalities occur in a minority of cases.8 ADEM is a rare inflammatory demyelinating condition of the central nervous system that can be triggered by viral infections and vaccinations. Immunosuppressive medications with or without plasma exchange is the mainstay of treatment. Most children with ADEM improve, with many achieving complete recovery. The prognosis of adults with ADEM is less favorable; permanent disability, recurrence, and death occur at higher rates in adults.9 Transverse myelitis is an acquired inflammatory disorder of the spinal cord that results in acute or subacute motor, sensory, and/or autonomic deficits below the level of the lesion. It is most often triggered by infection or vaccination, but can be associated with an underlying systemic disease (eg, systemic lupus erythematosus) or a demyelinating disease such as MS. Immunosuppressive medications with or without plasma exchange is the mainstay of treatment. The prognosis of transverse myelitis is highly variable. When associated with MS, patients may have complete or substantial recovery. When idiopathic or associated with other diseases, residual neurologic deficits are common. Most recovery occurs within the first 3 months after the initial event.10 The granulomatous inflammation of sarcoidosis can involve the spinal cord. There is usually evidence of systemic sarcoidosis in affected patients (the presence of hilar lymphadenopathy can be a clue). MRI of the spine can be suggestive of the diagnosis. CSF evaluation often reveals mononuclear pleocytosis with elevated protein and normal glucose. Ultimately, biopsy may be necessary to confirm the diagnosis. Sarcoid myelopathy is often responsive to treatment with glucocorticoids.2 A paraneoplastic syndrome is suggested by the detection of paraneoplastic antibodies in serum or CSF.
894
What is the most common Hepatic myelopathy is rare, usually occurring in patients with chronic liver disease and associated presentation of hepatic portosystemic shunt. The most common deficit is progressive spastic paraparesis. Prognosis may be myelopathy? improved with early recognition and treatment with liver transplantation.11 What hereditary conditions are Hereditary myelopathies predominantly involving UMNs include hereditary spastic paraplegias (HSP), associated with myelopathy adrenoleukodystrophy, and Friedreich’s ataxia.12 predominantly involving upper motor neurons? What is the timing of onset of Surfer’s myelopathy describes a nontraumatic myelopathy that develops when prolonged prone surfer’s myelopathy? positioning is followed by vigorous activity and assumption of the upright position. It is characterized by the onset of upper lumbar or thoracic pain, progressive paraparesis or paraplegia, and urinary retention, usually within an hour of surfing. It tends to affect novice surfers. Prognosis is variable; some experience full recovery and others are left with permanent paralysis.2 What is the mechanism of Caisson’s disease occurs when patients ascend too rapidly after exposure to high pressures under water. Caisson’s disease? Under these conditions, nitrogen bubbles form and become trapped in spinal vessels, resulting in ischemia. The thoracic spine is most frequently affected. Immediate recompression in a hyperbaric chamber is the treatment of choice. Some patients recover whereas others are left with permanent disability.2
895
Weakness Related to Lower Motor Neuron Lesions
What is a lower motor neuron? Lower motor neuron lesions occur within which 3 general anatomic structures?
What are the relationships between the anterior horn cells, spinal nerve roots, nerve plexuses, and peripheral nerves?
LMNs are motor neurons that reside in the brainstem and spinal cord and are controlled by UMNs. LMN lesions can involve the anterior horn cell, nerve root or plexus, or peripheral nerve.
Motor nerve fibers originating from the anterior horn cells of the spinal cord form the ventral nerve roots; neighboring roots join together to form plexuses that give rise to peripheral nerves (Figure 44-6).2
FIGURE 44-6 Spinal cord and spinal nerves. A, Posterior view showing nerve plexuses and some peripheral nerves. B, Cross-section of the spinal cord showing the organization of the gray and white matter. The roots of the spinal nerves are also shown. (From Cohen BJ, Hull KL. Memmler’s The Human Body in Health and Disease. 13th ed. Philadelphia, PA: Wolters Kluwer Health; 2015.)
896
897
Weakness Related to Lower Motor Neuron Lesions of the Anterior Horn Cell
What are the causes of Weakness Related to the anterior horn cell?
Lou Gehrig’s disease. Related to amyotrophic lateral sclerosis; this disease affects lower motor neurons only. An upper and lower motor neuron disorder of the cranial nerves. Related to amyotrophic lateral sclerosis; this motor neuron disease predominantly affects the upper extremities. Related to amyotrophic lateral sclerosis; this motor neuron disease predominantly affects the lower extremities. A mosquito-borne virus. Vaccination has almost eradicated this virus from parts of the world. Muscular atrophy is a prominent feature of this disease that predominantly occurs in children.
Does amyotrophic lateral sclerosis exclusively involve the anterior horn cell? What is the prognosis of progressive muscular atrophy? What is the prognosis of progressive bulbar palsy?
Amyotrophic lateral sclerosis (ALS). Progressive muscular atrophy (PMA).
Progressive bulbar palsy (PBP).
Flail arm syndrome (FAS).
Flail leg syndrome (FLS).
West Nile Virus. Poliomyelitis.
Spinal muscular atrophy (SMA).
ALS is a progressive neurodegenerative disorder that results in gradually worsening muscle weakness and eventual death. Degeneration involves the anterior horn cells as well as the corticobulbar and corticospinal tracts resulting in the characteristic combination of both lower and upper motor neuron signs. The disease is familial in around 10% of cases. Men are disproportionately affected by a ratio of 2:1. Onset is above 45 years of age in most cases. Primary lateral sclerosis is a condition closely related to ALS that only affects UMNs.2 PMA is more common in men by a ratio of 4:1. The clinical course tends to progress more slowly compared with ALS. The 5-year survival rate is approximately 70% in patients whose onset is before 50 years of age, and is around 40% in those diagnosed after 50 years of age.2 PBP is a disorder that first involves the upper and lower motor neurons of the brainstem, resulting in weakness of the cranial muscles (eg, jaw, face, tongue, pharynx, larynx). The prognosis of PBP is poor; most patients die within
898
What are some of the clinical differences between flail arm syndrome and amyotrophic lateral sclerosis? What are the clinical characteristics of flail leg syndrome? What are the characteristics of the weakness associated with West Nile virus infection? How is poliovirus spread in humans?
What other viral infections cause Weakness Related to involvement of anterior horn cells? What are the characteristics of adult-onset spinal muscular atrophy?
2 to 3 years of onset when weakness spreads to the respiratory muscles.2 Compared with ALS, FAS demonstrates an even stronger predominance in men, has a predilection for the proximal muscles of the upper extremities without significant weakness of the legs or bulbar sites, has less UMN involvement, and is associated with better prognosis.13
In patients with FLS, weakness and atrophy begin in the distal lower extremities and there is slow progression and subtle or late UMN involvement. Like FAS, FLS is associated with better prognosis compared with ALS.14 West Nile virus infection results in acute asymmetric flaccid paralysis, absent deep tendon reflexes, and preserved sensation. Although the major site of involvement is the anterior horn cell, other sites can be involved, including the adjacent white matter, dorsal root ganglia, and peripheral nerves.15
Poliovirus is mainly spread human-to-human, usually through the fecal-oral route. The vast majority of infections are either asymptomatic or associated with an influenza-like illness, whereas a small proportion involves the central nervous system, most typically causing acute, asymmetric, and flaccid weakness predominantly involving the lower extremities.16 Viruses that can involve anterior horn cells include non-polio enteroviruses (eg, enteroviruses D68 and 71), Japanese encephalitis virus, and HTLV-I.16
Adult-onset (type 4) SMA is an autosomal recessive disease that typically presents after 30 years of age, causing proximal limb and diaphragmatic weakness. It is slowly progressive and patients eventually become wheelchair bound but have normal life expectancy.2
899
Weakness Related to Lower Motor Neuron Lesions of the Root OR Plexus
What is a myotome? What are the characteristic manifestations of injury to the nerve root? Why is weakness caused by radiculopathy usually only mild in severity? What are the characteristic manifestations of injury to the nerve plexus? What are the 2 main plexopathies?
A myotome refers to the group of muscles innervated by a single spinal nerve root. Myotomes are helpful in localizing lesions of the neuromuscular system. Nerve root injury (ie, radiculopathy) usually results in weakness, pain, sensory loss, and diminished reflexes in the sensorimotor distribution of the involved nerve root (ie, the associated myotomes and dermatomes). If more than 1 root is involved, the term polyradiculopathy is used.2 Many muscles, particularly the larger ones, receive innervation from multiple nerve roots, which preserves strength in the setting of radiculopathy.2 Nerve plexus injury, also called plexopathy, results in motor, sensory, and reflex loss involving 1 extremity, but the patterns of deficits are often variable and complex.2 Brachial and lumbosacral plexopathies are the 2 main types. Lumbar and sacral plexuses are sometimes considered separate.
What are the causes of Weakness Related to the nerve root or plexus?
A 76-year-old man complains Lumbar spinal stenosis. of weakness and numbness of his legs when he stands or walks for prolonged periods of time; the symptoms resolve after he sits down. An Endocrinopathy Diabetes mellitus. commonly associated with polyneuropathy. A 47-year-old man with right Brachial plexopathy related to avulsion or stretch injury. arm weakness after a motorcycle accident. A 28-year-old woman Lyme disease. presents with bilateral facial droop, and right leg pain and weakness after a camping trip in Massachusetts. Misplacement of an epidural Arachnoiditis. injection could result in this chronic inflammatory process. A patient with hilar Sarcoidosis, complicated by cauda equina syndrome. adenopathy develops lower back pain, numbness of the perineum, bladder and bowel incontinence, and Decreased anal sphincter tone. A patient is awoken in the Neuralgic amyotrophy (ie, Parsonage-Turner syndrome). middle of the night with sudden-onset severe “shooting” pain that radiates from the right shoulder to the arm, which is followed a few days later by weakness of the right arm, and is found to have a “winged” scapula.
900
What are the compressive causes of neuropathy involving the nerve root or plexus? What physical examination maneuver is highly specific for compressive cervical radiculopathy? What are the characteristics of diabetic lumbosacral radiculoplexus neuropathy?
What are the iatrogenic causes of traumatic root or plexus injury? What are the infectious causes of root or plexus pathology? What imaging finding is associated with arachnoiditis? What are the characteristics of sarcoid radiculopathy?
What is neuralgic amyotrophy?
Compressive causes of neuropathy involving the nerve root or plexus include degenerative disease (eg, osteoarthritis), spondylolisthesis, spinal stenosis, disc herniation, benign or malignant tumor, syringomyelia, epidural abscess, hematoma, and thoracic outlet obstruction. Spurling test is specific for compressive cervical radiculopathy. The patient’s head is extended and rotated toward the side of the pain, then downward pressure is applied on the top of the head. The test is positive if symptoms in the arm are reproduced in a radicular distribution.17 Diabetic patients can develop a variety of disorders of the peripheral nervous system. Diabetic lumbosacral radiculoplexus neuropathy (ie, diabetic amyotrophy) refers to a distinct neuropathy that involves the lumbosacral radiculoplexus. Affected patients tend to be older with relatively well-controlled or undiagnosed diabetes. Pain starts in the low back or hip and spreads unilaterally to the thigh and knee. It is described as deep and aching and tends to be worse at night. Later, weakness and atrophy develop in the pelvic girdle and thigh muscles. The pain generally begins to subside spontaneously after several days. Motor recovery virtually always occurs, but usually requires months to years.2 Iatrogenesis accounts for up to 10% of brachial plexopathies. Root or plexus injury can occur during surgery (from direct injury or surgical positioning), attempted reduction of shoulder dislocations, anesthetic regional blocks, and radiation therapy.18 Infectious causes of root or plexus pathology include Mycobacterium tuberculosis (ie, Pott’s disease), varicella-zoster virus, cytomegalovirus, syphilis, Lyme disease, schistosomiasis, and strongyloides.1 In patients with arachnoiditis, inflammation and subsequent proliferation of connective tissue results in thickening of the arachnoid membrane and eventual obliteration of the subarachnoid space, which is evident on imaging of the spine.2 Sarcoidosis of the peripheral nervous system most commonly involves the cranial nerves (eg, facial nerve palsy). Polyradiculopathy, which is less common, primarily involves the thoracic and lumbar roots and may improve with systemic glucocorticoid therapy. Laminectomy may be considered for refractory cases.19 Neuralgic amyotrophy (ie, Parsonage-Turner syndrome) is an acquired clinical syndrome characterized by episodes of neuropathic pain and patchy paresis of the upper extremities. It most often involves the upper part of the brachial plexus, resulting in weakness of the infraspinatus and serratus anterior muscles (leading to a winged scapula). The pathophysiology of this condition is incompletely understood but is thought to be immune-mediated. Most patients recover over the course of a few years, but some will experience chronic pain and motor dysfunction. A rare familial form exists, known as hereditary neuralgic amyotrophy. Patients with inherited neuralgic amyotrophy tend to be younger, experience more frequent recurrent attacks, and are more likely to develop nerve involvement outside the brachial plexus.20
901
Weakness Related to Lower Motor Neuron Lesions of the Peripheral Nerve
What are the causes of Weakness Related to the peripheral nerve?
A single peripheral nerve is affected. At least 2 noncontiguous individual nerves are affected. Usually characterized by a generalized neuropathy affecting many peripheral nerves in a symmetric, length-dependent fashion.
Mononeuropathy. Mononeuritis multiplex.
Polyneuropathy.
What physical Mononeuropathy involving the median nerve (ie, carpal tunnel syndrome) is characterized by the presence of findings are present neuropathic symptoms and signs along the distribution of the median nerve (ie, involving the thumb, index, in the setting of middle finger, and the radial side of the ring finger). Tinel’s test (tapping the volar surface of the wrist) or Phalen’s mononeuropathy test (flexion of the wrist for >60 seconds) may be positive for pain or paresthesias in the distribution of the median involving the median nerve. Atrophy of the thenar eminence and weakness and atrophy of the abductor pollicis brevis and opponens nerve? pollicis become evident in advanced cases.21 What are the causes Mononeuritis multiplex describes the presence of neuropathy involving 2 or more noncontiguous individual of mononeuritis nerves. More than one-half of all cases are caused by involvement of the vasa nervorum by a systemic vasculitis multiplex? such as polyarteritis nodosa. Other causes include diabetes mellitus, Lyme disease, sarcoidosis, HIV infection, and leprosy.2 P For a detailed discussion of polyneuropathy, see chapter 41, olyneuropathy.
902
PA: Wolters Kluwer Health; 2017.)
Weakness Related to Disorders of the Neuromuscular Junction
What is the neuromuscular junction?
The NMJ is the interface between a single nerve fiber and its corresponding muscle fiber; it is the junction where electrical activity of the nerve is translated into muscle contraction (Figure 44-7).2
FIGURE 44-7 Relationship between the upper motor neuron, lower motor neuron, neuromuscular
Which molecule is primarily involved in the communication between nerve and muscle? What triggers the release of acetylcholine from the nerve terminal to the synaptic cleft? How does acetylcholine stimulate muscle action?
What enzyme is involved in hydrolyzing bound acetylcholine?
junction, and muscle. A, Detail of the neuromuscular junction. B, Detail of skeletal muscle. (Adapted from McConnell TH. The Nature of Disease Pathology for the Health Professions, 2nd ed. Philadelphia,
Acetylcholine (ACh) is the primary neurotransmitter of the NMJ.2
The arrival of an axonal action potential triggers several steps that ultimately result in the release of ACh into the synaptic cleft via an exocytotic process.2 After ACh is released into the synaptic cleft, it binds to receptors on the postsynaptic membrane, which triggers depolarization, entry of calcium ions, and generation of an action potential in the muscle membrane, leading to contraction of the muscle.2 ACh is hydrolyzed by acetylcholinesterase. This serves to terminate the action potential, allowing for sequential muscle activation.2
What are the causes of Weakness Related to the neuromuscular junction?
A young Myasthenia gravis associated with thymoma. woman presents with weakness that worsens toward the end of the day and is found to have a mediastinal mass. A 74-year-old Lambert-Eaton syndrome. man with small cell lung cancer presents with fluctuating weakness.
903
snakes, lizards, spiders, scorpions, insecticides. plants, and
What physical Myasthenia gravis is the most common disorder of the NMJ. It is an autoimmune condition in which ACh receptors are finding is blocked or destroyed, resulting in impaired neuromuscular transmission. The hallmark clinical manifestation of characteristic of myasthenia gravis is weakness that presents or worsens with repetitive activity (ie, fatigability). The muscles of the eyes, myasthenia face, jaw, throat, and neck are often the first to be affected, resulting in manifestations such as diplopia, ptosis, gravis? dysphagia, difficulty chewing, and dysarthria. Although the course can be variable, disease activity typically begins intermittently and becomes more persistent over time. Pharmacologic management includes anticholinesterases and immunosuppressants (eg, glucocorticoids). Other therapeutic modalities include plasma exchange and thymectomy.2 What physical Lambert-Eaton syndrome is a disorder of the NMJ that is strongly associated with small cell lung cancer. It is caused by finding is the presence of autoantibodies that result in the functional loss of voltage-gated calcium channels on the presynaptic characteristic of motor nerve terminal. Muscles of the trunk, shoulder girdle, pelvic girdle, and lower extremities are disproportionately Lambert-Eaton affected. Weakness that improves with repetition (the opposite of myasthenia gravis) is characteristic of Lambert-Eaton syndrome? syndrome.2 What are the Within 12 to 36 hours of exposure to botulinum toxin (usually by ingestion), patients experience nausea, vomiting, and typical clinical anorexia. The initial neurologic manifestations include blurred vision and diplopia related to ptosis, strabismus, and manifestations extraocular muscle palsies. This is followed quickly by other bulbar manifestations including voice changes, dysarthria, of botulism? and dysphagia. Progressive weakness of the face, neck, trunk, and extremities are usually the last manifestations.2 Which ticks are Ticks that cause NMJ dysfunction vary by region. For example, in Canada and the northwestern United States, the most most commonly common offender is the wood tick Dermacentor andersoni, whereas in the southeastern United States, it is the dog tick responsible for Dermacentor variabilis.2 dysfunction of the neuromuscular junction? Which spider is Venom from the black widow spider ( ) can cause NMJ dysfunction.2 most commonly Latrodectus associated with dysfunction of the neuromuscular junction? Which Exposure to organophosphates and carbamates can result in NMJ dysfunction.22 insecticides are most commonly associated with dysfunction of the neuromuscular junction?
904
Weakness Related to MyopathY
What is a muscle fiber? Muscle fibers are multinucleated cells that vary in length and diameter. Each muscle fiber receives innervation from a terminal branch of an axon originating from an anterior horn cell in the spinal cord or the motor nuclei of a cranial nerve in the brainstem. A single muscle is composed of thousands of fibers (see Figure 44-7).2 What are the most common Symptoms of myopathy may include weakness (most frequent), pain, spasm, cramping, twitching, symptoms and signs of myopathy? myotonia, and a change in muscle size (usually atrophy).2 Which electrolyte is particularly The regulation of muscle contraction is principally dependent on calcium.2 important in regulating muscle contraction? What are the 2 main contractile Actin and myosin are the main contractile proteins in muscle.2 proteins in muscle? What is the source of chemical Adenosine triphosphate (ATP) provides the chemical energy for muscle contraction.2 energy for muscle contraction? What enzyme, found in high Creatine kinase (CK) can be found in high concentrations in the serum of patients with myopathy.2 concentrations within muscle cells, can be an important serologic marker of myopathy?
What are the myopathic causes of weakness?
A 47-year-old woman presents with lower extremity discomfort and weakness a few weeks after starting a medication for hypercholesterolemia. The elevation of aspartate aminotransferase and alanine aminotransferase in a ratio >2:1 may be a clue to the underlying diagnosis. An electrolyte disturbance associated with primary hyperaldosteronism (ie, Conn’s syndrome). Prolonged immobility. A 50-year-old woman with central obesity, abdominal striae, and progressive proximal muscle weakness. A condition often associated with the intensive care unit. Following a tropical storm in Thailand, a young woman develops cola-colored urine and weakness after being removed from her damaged home. A patient develops strabismus, diplopia, and dysarthria after consuming undercooked pork. An immune-mediated condition. A progressive hereditary degenerative muscle disease. Disorders of the breakdown, use, and storage of the sources of energy for muscle contraction.
Stain-induced myopathy.
Alcohol-induced myopathy.
Hypokalemia.
Deconditioning. Cushing’s syndrome.
Critical illness myopathy.
Rhabdomyolysis.
Trichinosis.
Inflammatory myopathy. Muscular dystrophy.
Metabolic myopathy.
905
What are the clinical hallmarks of medication-induced myopathy?
What toxins are associated with myopathy? Which electrolyte disturbances are associated with myopathy? What is the timing of onset of deconditioning during the course of inactivity? Which endocrinopathies can cause myopathy? How can critical illness polyneuropathy be distinguished from critical illness myopathy? What is rhabdomyolysis? What are the infectious causes of myopathy? What are the inflammatory myopathies? How can a handshake provide a clue to the diagnosis of myotonic dystrophy? Which energy substrates are necessary for the sustenance of normal muscular activity?
Medication-induced myopathy typically occurs in patients without preexisting muscular symptoms. It is suggested by a delay in the development of symptoms after exposure to a causative agent and the complete or partial improvement of symptoms after withdrawal of the agent. The absence of an alternative cause of myopathy is additionally supportive. The prognosis of medication-induced myopathy is variable, mirroring the wide-ranging mechanisms and degrees of severity of the condition. However, most cases completely or at least partially resolve after discontinuation of the offending agent.2 Toxins associated with myopathy include alcohol, cocaine, amphetamines, heroin, mushroom poisoning (eg, Amanita phalloides), and exogenous or endogenous glucocorticoids.2 Numerous electrolyte disturbances are associated with myopathy, including hyper/hyponatremia, hyper/hypokalemia, hypophosphatemia, hypocalcemia, and hypomagnesemia.23
Deconditioning occurs within days to weeks following a sudden decrease in activity. It appears to be primarily mediated through a downregulation of protein synthesis resulting in loss of muscle mass.24
Endocrinopathies associated with myopathy include hypothyroidism, hyperthyroidism, Cushing’s syndrome, acromegaly, and others that are associated with electrolyte disturbances (eg, primary hyperaldosteronism).2 Electromyography can be used to localize a lesion and distinguish neuropathy from myopathy. In some cases, both conditions may be present. Most cases of critical illness myopathy develop in association with the administration of high doses of glucocorticoids, but it can occur in patients with sepsis and shock who are not exposed to steroids.2 Rhabdomyolysis occurs when there is rapid skeletal muscle breakdown (eg, from a crush injury), manifesting with painful and sometimes weak muscles, elevated serum CK, myoglobinuria, and Acute Kidney Injury.2 Infectious causes of myopathy include parasites (eg, trichinosis, toxoplasmosis), viruses (eg, HIV, HTLV-I, influenza), bacteria (eg, Staphylococcus aureus), and rarely, fungi.25 The main 3 inflammatory myopathies are dermatomyositis (DM), polymyositis (PM), and inclusion body myositis (IBM). Others include eosinophilic myositis, vasculitis, immune-mediated necrotizing myopathy, granulomatous myositis (eg, sarcoidosis), myositis related to graft-versus-host disease, and myositis related to connective tissue disease (eg, systemic lupus erythematosus). Myotonic dystrophy, which is the most common adult muscular dystrophy, is associated with prolonged failure of relaxation after muscle contraction, called myotonia. This delayed relaxation can be appreciated on shaking hands with an affected individual.2 During early exercise, glucose is the main source of energy; when glycogen stores become depleted, the principle source of energy comes from the oxidation of fatty acids.2
906 Case Summary wA 52-year-old man presents with progressive proximal muscle What is the most likely cause of weakness in this patient? Dermatomyositis. eakness and is found to have abnormal skin findings.
907
Bonus Questions
difference between dermatomyositis What is the significance of the skin What is the primary phenotypic and polymyositis? findings in this case?
dermatitis relative to myositis in What is the timing of onset of patients with dermatomyositis? What systemic conditions are
associated withidermatomyositis and polymyosit s? Which malignancies are most often polymyositis? linked to dermatomyositis and What are the epidemiologic differences between the 3 main (dermatomyositis, polymyositis, and inflammatory myopathies inclusion body myositis)? What laboratory tests can be helpful
in the diagnosis of the inflammatory myopathies, including dermatomyositis? What supplementary tests can be helpful in the diagnosis of the inflammatory myopathies? patients with acute inflammatoryr What is the treatment of choice fo myopathy? What is the prognosis of
inflammatory myopathy?
DM is characterized by involvement of the skin (ie, dermatitis).2
The discoloration around the patient’steyes in this case (see Figure 44-1) is known as a “heliotrope” rash (ie, lilac-colored change in the skin over the eyelids and around the eyes, some imes with edema). The violaceous papules over the dorsal joints of the hands (see Figure 44-2) are known as Gottron’s papules. Other skin findings characteristic of DM include an erythematous rash over the neck and chest, known as the “V” sign (when present over the shoulders and upper arms, it is known as the “shawl” sign), cutaneous calcifications, and erythroderma.2 The skin findings of DM typically precede the muscle findings.2
DM and PM are often associated with connective tissue diseases (eg, rheumatoid arthritis), malignancy, and other autoimmune conditions (eg, myasthenia gravis). Antisynthetase syndrome occurs in some patients with DM or PM and is characterized by a constellation of clinical manifestations, including fever, interstitial lung disease, inflammatory myopathy, polyarticular inflammatory arthritis, Raynaud’s phenomenon, and mechanic’s hands (see ). Figure 47-4 2,26 Underlying malignancies most often associated with DM and PM in men are those of the lung and colon; in women, breast and ovarian malignancies are most common. 2 patients with PMtare adults (30-60 years of age), and there is a propensity for women; IBM occurs most commonly in patientsiover 50;years of age, DM equally affec s adults and children with a propensity for females in adulthood but an equal propensity for the sexes in ch ldhood most and there is a 3:1 predominance of males:females. 2
alsoibe present in patients with PM or DM, particularly in,the setting of an underlying connective tissue disease (eg, anti-histidyl-tRNA synthetase EMG, MRI, and biopsy of the muscle are often helpful in establishing the diagnosis of inflammatory myopathy.2 Serum levels of muscle enzymes, such as CK and aldolase are often elevated in patients with inflammatory myopathy. Various antibodies may [ant -Jo-1] in patients with antisynthetase syndrome). 2
Systemic glucocorticoids are considered first-line treatment for acute PM and DM. IBM does not consistently respond to treatment with glucocorticoids. 2
malignancy or connective tissue disease. IBM typically progresses over many years, sometimes quite slowly, but most patients become disabled PM and DM are typically responsive to glucocorticoids and prognosis is generally favorable, except in cases associated with underlying over time. 2
908 Key Points
Weakness is defined as a reduction in power generated,by muscle. Weakness can be caused by disorders of the upper motor neuron, predominantlylinvolve the extensors and abductorslof the upper UMN weakness is associated with the following physical findings: (spasticity), hyperreflexia, and positive Babinski reflex. Fatigue and asthenia are often confused with weakness but sometimes these conditions coexist. lower motor neuron, neuromuscular junction, or muscle. UMN lesions occur proximal to the anterior horn ce l and limbs and the f exors of the lower limbs, and affect proximal > distal muscles. little to no atrophy, absence of fasciculations, Increased tone UMN lesions can involve the brain or spinal cord. LMN lesions involve the structures distal to and including the LMN weakness is associated with the following physical findings: anterior horn cell and affect distal >proximal muscles. severe atrophy, fasciculations, diminished tone, diminished reflexes, and absent Babinski reflex.
LMN lesions can involve the anterior horn cell, nerve root or NMJ disorders presentfwith variable distributions of weakness.h Myopathic weakness tends to be symmetric and affect Myopathic weakness is associated with the following physical diminished tone, normal or diminished reflexes, and absent plexus, or peripheral nerve. The prototypic cause o NMJ disease is myasthenia gravis, whic is characterized by fatigable weakness. proximal >distal muscles. findings: mild atrophy, absence of fasciculations, normal or Babinski reflex.
909
References 1. Longo DL, Fauci AS, Kasper DL, Hauser SL, Jameson JL, Loscalzo J, eds. Harrison’s Principles of Internal Medicine. 18th ed. New York, NY: McGraw-Hill; 2012.
-
Ropper AH, Samuels MA, Klein JP, eds. Adam and Victor’s Principles of Neurology. 10th ed. China: McGraw-Hill Education; 2014.
-
Sapira JD. The Art & Science of Bedside Diagnosis. Baltimore, Maryland, USA: Urban and Schwarzenberg; 1990.
-
HabektM. Evaluation of brainstem involvement in multiple sclerosis. Expert Rev 5. Alexander.M, Murthy JM. Acute disseminated encephalomyelitis: treatment Neuro her. 2013;13(3):299-311. guidelines Ann Indian Acad Neurol. 2011;14(suppl 1): S60-S64.
-
Devivo MJ. Epidemiology of traumatic spinal cord injury: trends and future implications. Spinal Cord. 2012;50(5):365-372.
-
Berger JR, Sabet A. Infectious myelopathies. Semin Neurol. 2002;22(2):133-142.
- Bot JC, Barkhof F, Polman CH, et al. Spinal cord abnormalities in.recently(diagnosed 9. Ketelslegers IA, Visser IE, Neuteboom RF, Boon M, Catsman-Berrevoets CE, Hintzen MS patients: added value of spinal MRI examination. Neurology 2004;62 2):226-233. RQ. Disease course and outcome of acute disseminated encephalomyelitis is more
severe in adults than in children. Mult Scler. 2011;17(4):441-448. 10. Frohman EM, Wingerchuk DM. Clinical practice. Transverse myelitis. N Engl J Med. 2010;363(6):564-572.
- Nardone R, Buratti T, Oliviero A, Lochmann A, Tezzon F. Corticospinal involvement in patients with a portosystemic shunt due to liver cirrhosis: a MEP study. J Neurol.
2006;253(1):81-85. 12. Ginsberg L. Disorders of the spinal cord and roots. Pract Neurol. 2011;11(4):259-267. 13flail arm syndrome and amyotrophic lateral sclerosis. PLoS One. 2015;10(6):e0127601. 14. Wijesekera LC, Mathers S, Talman P, et al. Natural history and clinical features of the . Yang H, Liu M, Li X, Cui B, Fang J, Cui L. Neurophysiological differences between flail arm and flail leg ALS variants. Neurology. 2009;72(12):1087-1094.
-
Leis AA, Stokic DS. Neuromuscular manifestations of west nile virus infection. Front Neurol. 2012;3:37.
-
Howard RS. Poliomyelitis and the postpolio syndrome. BMJ. 2005;330(7503):1314- 1318.
-
Caridi JM, Pumberger M, Hughes AP. Cervical radiculopathy: a review. HSS J. 2011;7(3):265-272.
-
Wilbourn AJ. Iatrogenic nerve injuries. Neurol Clin. 1998;16(1):55-82.
-
Koffman B, Junck L, Elias SB, Feit HW, Levine SR. Polyradiculopathy in sarcoidosis. Muscle Nerve. 1999;22(5):608-613.
-
van Alfen N, van Engelen BG. The clinical spectrum of neuralgic amyotrophy in 246 cases. Brain. 2006;129(Pt 2):438-450.
-
Ibrahim I, Khan WS, Goddard N, Smitham P. Carpal tunnel syndrome: a review of the recent literature. Open Orthop J. 2012;6:69-76.
-
Jokanovic M. Medical treatment of acute poisoning with organophosphorus and carbamate pesticides. Toxicol Lett. 2009;190(2): 107-115.
-
Yu J. Endocrine disorders and the neurologic manifestations. Ann Pediatr Endocrinol
910
Metab. 2014;19(4):184-190. 24. Berry MJ, Morris PE. Early exercise rehabilitation of muscle weakness in acute respiratory failure patients. Exerc Sport Sci Rev. 2013;41(4):208-215.
-
Crum-Cianflone NF. Bacterial, fungal, parasitic, and viral myositis. Clin Microbiol Rev. 2008;21(3):473-494.
-
Katzap E, Barilla-LaBarca ML, Marder G. Antisynthetase syndrome. Curr Rheumatol Rep. 2011;13(3):175-181.
911